Final Exam

अब Quizwiz के साथ अपने होमवर्क और परीक्षाओं को एस करें!

A 50-year-old woman sees her physician because of a 3-month history of fatigue on exertion and dizziness. She has recently experienced syncopal episodes. She has trouble keeping her eyes open toward the end of the day and has double vision. She has no muscle aches or pains. On physical examination, she is afebrile, with a pulse of 88/min, respirations of 19/min, and blood pressure of 115/75 mm Hg. She exhibits marked pallor, but no hepatosplenomegaly or lymphadenopathy. There is decreased motor strength with repetitive motion, but no apparent muscle atrophy, joint deformity, pain, or redness. Laboratory findings show hemoglobin of 6.6 g/dL, hematocrit of 19.9%, platelet count of 199,800/mm3, WBC count of 4780/mm3, and reticulocyte count of 0.1%. The MCV, MCHC, and serum ferritin level are normal. A bone marrow biopsy specimen of aspirate shows normal cellularity, but the cells of the erythroid series, such as pronormoblasts, normoblasts, and later stages, are greatly reduced. Other elements are normal in number and differentiation. The figure shows a chest CT scan. Which of the following serologic laboratory tests would be most helpful in diagnosing her underlying condition? □ (A) Acetylcholine receptor antibody □ (B) Anti-DNA topoisomerase □ (C) Anti-glomerular basement membrane antibody □ (D) Antimitochondrial antibody □ (E) Antinuclear antibody

Acetylcholine receptor antibody (findings describe myasthenia gravis with complications of thymoma and RBC aplasia; thymic disorders are common in myasthenia gravis; Ab against acetylcholine receptor disrupt the function of the myoneural junction)

A 73-year-old man has had bilateral knee and hip pain for the past 25 years and has taken a medication for this pain for the past 5 years. During the past year, he has noticed increasing frequency of headaches, dizziness, tinnitus, confusion, and nausea. He states that bruises form on his skin with minimal trauma. One week ago, he experienced an episode of hematemesis. On physical examination, his temperature is 37.1°C, pulse is 73/min, respirations are 18/min, and blood pressure is 130/80 mm Hg. There are scattered petechiae on his arms and legs and an area of purpura on his right thigh. His heart rate is regular, and the lungs are clear. No neurologic deficits are noted. Laboratory findings show hemoglobin, 11.1 g/dL; hematocrit, 33.1%; MCV, 72 μm3; platelet count, 317,200/mm3; WBC count, 5915/mm3; Na+, 139 mmol/L; K+, 3.9 mmol/L; Cl−, 98 mmol/L; CO2, 19 mmol/L; glucose, 76 mg/dL; and creatinine, 1.1 mg/dL. The partial thromboplastin time is 27 seconds, and the prothrombin time is 13 seconds. Platelet function analysis shows decreased aggregation in response to ADP and collagen stimulation. An upper gastrointestinal endoscopy shows gastric mucosal erythema and a 1.8-cm, sharply demarcated, shallow, antral ulceration. Long-term use of which of the following pharmacologic agents is most likely to produce these findings? □ (A) Acetylsalicylic acid □ (B) Acetaminophen □ (C) Adalimumab □ (D) Methotrexate □ (E) Oxycodone □ (F) Propoxyphene

Acetylsalicylic acid (metabolic acidosis, tinnitus, platelet function abnormalities, and gastritis with ulceration are typical effects of excessive aspirin ingestion)

A translational research project is focused on development of a pharmacologic agent that would affect molecular signaling pathways within cells. This agent binds to type 1 TNF receptor (TNFR1), which triggers activation of intracellular caspases. For what condition is this agent most likely to be useful? □ (A) Adenocarcinoma □ (B) Atherosclerosis □ (C) Autoimmunity □ (D) Cirrhosis □ (E) Dementia □ (F) Osteoporosis

Adenocarcinoma (mechanism described is that of apoptosis; apoptosis in tumors would cause self-destruction)

42-year-old woman has had increasing weakness, nausea, vomiting, watery diarrhea, and a 5-kg weight loss over the past 7 months. She has generalized muscle weakness, muscle wasting, and increased skin pigmentation on physical examination. After an upper respiratory tract infection lasting 1 week, she develops abdominal pain and faintness and lapses into a coma. On admission to the hospital, her temperature is 36.9°C, pulse is 83/min, respirations are 17/min and shallow, and blood pressure is 80/40 mm Hg. Laboratory findings show hemoglobin, 13.6 g/dL; hematocrit, 43.8%; WBC count, 5420/mm3; Na+, 129 mmol/L; K+, 3.5 mmol/L; Cl−, 95 mmol/L; CO2, 23 mmol/L; glucose, 48 mg/dL; and creatinine, 0.6 mg/dL. Atrophy of which of the following tissues is most likely to be present? □ (A) Adrenal cortex □ (B) Islets of Langerhans □ (C) Hypothalamus □ (D) Parafollicular cells □ (E) Pineal gland □ (F) Thyroid epithelium

Adrenal cortex (patient has Addison disease with bilateral atrophy of adrenal cortex; most often idiopathic; leads to electrolyte changes owing to loss of mineralcorticoid secretion, mainly aldosterone; marked atrophy leads to decreased gluticocorticoids, which increases ACTH precursor hormones and stimulates skin melanocytes)

A 24-year-old, previously healthy man who smokes one pack of cigarettes per day and who works as a histotechnologist has developed a cough with bloody sputum over the past 2 days. He has increasing lethargy and nausea. A chest radiograph shows diffuse infiltrates most pronounced in the lower lobes. Laboratory findings show hemoglobin, 13.7 g/dL; hematocrit, 40.6%; MCV, 91 μm3; platelet count, 361,000/mm3; WBC count, 7385/mm3; Na+, 144 mmol/L; K+, 4.3 mmol/L; Cl−, 103 mmol/L; CO2, 26 mmol/L; creatinine, 3.8 mg/dL; urea nitrogen, 36 mg/dL; and glucose, 75 mg/dL. An abdominal ultrasound scan shows normal-sized kidneys. A renal biopsy specimen shows a crescentic glomerulonephritis. Which of the following mechanisms most likely produced this patient's pulmonary disease? □ (A) Antibody directed against basement membrane collagen □ (B) Apoptosis induced by CD8+ lymphocytes □ (C) Complement activation by circulating antigen-antibody complexes □ (D) Macrophage activation by CD4+ lymphocytes □ (E) Release of inflammatory mediators from mast cells

Antibody directed against basement membrane collagen (patient has findings of Goodpasture syndrome; antibody directed against the glomerular BM; also acts on BM in lung to produce pulmonary hemorrhage and hemoptysis)

A 43-year-old woman has had increasing difficulty swallowing over the past year. She notices that her hands turn white and are painful on exposure to cold. She remarks, "I may be getting older, but at least I don't have any wrinkles on my face or hands yet." On physical examination, her temperature is 37°C, pulse is 68/min, respirations are 14/min, and blood pressure is 115/75 mm Hg. The skin of her face and hands appears taut and shiny. A punch biopsy specimen of the skin of the hand shows dermal collagenous fibrosis and focal calcification. She receives yearly esophageal dilation for the next 20 years, during which time she develops no serious illnesses. Which of the following serologic test results is most likely to be positive? □ (A) Anticentromere antibody □ (B) Anti-DNA topoisomerase antibody □ (C) Antigliadin antibody □ (D) Antimicrosomal antibody □ (E) Antimitochondrial antibody □ (F) ANCA

Anticentromere antibody (patient has features of CREST syndrome or limited scleroderma; since disease has not progressed to serious pulmonary fibrosis or renal disease, she is less likely to have diffuse scleroderma that is associated iwth anti-DNA topoisomerase antibody)

A 34-year-old woman has had increasing lethargy for the past 8 months. During this time, she has experienced increased sensitivity to sunlight, and now rarely goes outdoors during the day. She has pain in her hands, elbows, knees, and feet, and muscle aches in her arms and legs. She has had increasing dyspnea for the past week. Physical examination shows no joint deformities, swelling, or redness. On auscultation of the chest, a friction rub is audible. A chest radiograph shows bilateral pleural effusions. Laboratory findings show hemoglobin, 11.6 mg/dL; hematocrit, 34.3%; MCV, 84 μm3; platelet count, 133,400/mm3; WBC count, 4610/mm3; Na+, 140 mmol/L; K+, 4 mmol/L; Cl−, 99 mmol/L; CO2, 25 mmol/L; glucose, 80 mg/dL; creatinine, 2.4 mg/dL; and calcium, 7.9 mg/dL. Which of the following additional laboratory tests would be most helpful to diagnose her underlying condition? □ (A) Acetylcholine receptor antibody □ (B) Anti-DNA topoisomerase antibody □ (C) Anti-glomerular basement membrane antibody □ (D) Antimicrosomal antibody □ (E) Antimitochondrial antibody □ (F) Antinuclear antibody

Antinuclear antibody (patient has findings consistent with SLE: photosensitivity, renal failure, body cavity effusions, pericarditis, arthralgias, myalgias, and cytopenias; ANA test is the most specific test for SLE)

A 33-year-old woman, G3, P0, who has had two spontaneous abortions, is in the second trimester of her third pregnancy. An ultrasound at 18 weeks' gestation revealed symmetric growth retardation. She gives birth to a stillborn fetus at 25 weeks, and experiences sudden onset of dyspnea. A pulmonary ventilation/perfusion scan indicates a high probability of thromboembolism. Four months later, she experiences an altered state of consciousness and sudden loss of movement in the right arm. A cerebral angiogram shows occlusion of a branch of the left middle cerebral artery. Laboratory findings show hemoglobin, 13.4 g/dL; hematocrit, 40.3%; MCV, 91 μm3; platelet count, 124,000/mm3; WBC count, 5530/mm3; prothrombin time, 13 seconds; partial thromboplastin time, 46 seconds; positive anticardiolipin antibody; positive serologic test result for syphilis; and negative ANA. Which of the following best explains these findings? □ (A) Antiphospholipid syndrome □ (B) Myeloproliferative disorder □ (C) Thrombophlebitis □ (D) Treponema pallidum infection □ (E) Trousseau syndrome □ (F) Von Willebrand disease

Antiphospholipid syndrome (some patients with this syndrome have systemic lupus erythematosus, but others do not - this is the case with this patient; arterial and deep venous thrombosis can occur)

A 57-year-old woman experiences a sudden loss of consciousness. She has a history of untreated hypertension, and has smoked one pack of cigarettes per day for the past 40 years. On physical examination, her temperature is 37.1°C, pulse is 70/min and irregular, respirations are 18/min, and blood pressure is 90/40 mm Hg. Carotid and radial pulses are diminished compared with femoral and posterior tibial pulses. Auscultation of the chest reveals faint heart sounds; lung fields are clear. A chest radiograph shows a widened mediastinum. The chest CT scan is shown in the figure. Pericardiocentesis is performed, and there is blood in the aspirate. What condition is most likely to produce these findings? □ (A) Aortic dissection □ (B) Bicuspid aortic valve □ (C) Small-cell anaplastic carcinoma □ (D) Takayasu arteritis □ (E) Tertiary syphilis □ (F) Thromboangiitis obliterans

Aortic dissection (extends proximally to envelop and partially occlude the great vessels; blood has moved into the mediastinum, causing widening, and into the pericardial cavity, causing tamponade)

A 61-year-old man has become more withdrawn and less active over the past 2 years. He spends most of his day in bed, although he seems to have minimal difficulty with movement. He has become less talkative. On physical examination, he has 5/5 motor strength in all extremities; there is no apparent tremor or ataxia. There are no focal neurologic deficits. He can remember only one of three objects after 3 minutes. His mood is depressed. His condition improves with use of Ginkgo biloba extract. One year later, he has an episode of aspiration while eating and dies 1 week later of pneumonia. At autopsy, the brain weighs 1000 g. The gross appearance is shown in the figure. Which of the following microscopic findings is most likely to be seen in the frontal cortex? □ (A) Spongiform change □ (B) Intranuclear inclusions □ (C) Alzheimer type II cells □ (D) Aβ amyloid deposits □ (E) Arteriolosclerosis □ (F) Loss of pigmented neurons □ (G) Absence of Betz cells

Aβ amyloid deposits (findings are characteristic of Alzheimer's disease; progressive over several years; brain is decreased in size, with narrow gyri and widened sulci in all lobes except the occipital; microscopically has neuritic plaques and Aβ amyloid cores)

A 58-year-old man reports increasing malaise over the past 8 months. He recently developed a cough productive of yellowish sputum, and a sputum culture grew Streptococcus pneumoniae. On physical examination, he has no lymphadenopathy or hepatosplenomegaly. Laboratory studies show serum creatinine, 4 mg/dL; urea nitrogen, 38 mg/dL; total protein, 9.3 g/dL; albumin, 4.1 g/dL; and alkaline phosphatase, 297 U/L. The finding on serum protein electrophoresis is shown in the figure. Which of the following laboratory findings is most likely to be reported? □ (A) TdT+ circulating blasts □ (B) Bence Jones proteinuria □ (C) Bone marrow karyotype with t(8;14) □ (D) Reactive amyloidosis □ (E) Hematocrit of 62%

Bence Jones proteinuria (multiple myeloma compromises abnormal plasma cells that tend to retain the ability to secrete Ig; light chains are excreted in urine and called Bence Jones proteins)

A 4-year-old girl has become increasingly listless over the past year. The child is at the 25th percentile for height and weight. On physical examination, there is pubic hair and clitoral and breast enlargement. There is no hepatomegaly, splenomegaly, or lymphadenopathy. The neurologic examination is unremarkable. Laboratory findings show hemoglobin, 13.7 g/dL; hematocrit, 41.8%; WBC count, 7120/mm3; Na+, 128 mmol/L; K+, 4.8 mmol/L; Cl−, 99 mmol/L; CO2, 21 mmol/L; glucose, 69 mg/dL; and creatinine, 0.5 mg/dL, and ACTH of 95 pg/mL with loss of diurnal rhythm of secretion. What disease process is most likely associated with these findings? □ (A) Anaplastic carcinoma of the thyroid □ (B) Bilateral adrenal hyperplasia □ (C) Islet cell adenoma □ (D) Medullary carcinoma of the thyroid □ (E) Neuroblastoma of the adrenal □ (F) Pituitary microadenoma □ (G) Suprasellar craniopharyngioma

Bilateral adrenal hyperplasia (findings are consistent with adrenogenital syndrome characterized by 21-hydroxylase deficiency and salt wasting; cortisol synthesis is blocked and ACTH secretion is increased, which stimulates androgen production)

In an experiment, a nuclear chromosomal gene is found to be actively transcribing mRNA that is transported into the cell cytoplasm. However, there is no observed protein product from translation of this mRNA. How is the silencing of this active gene's mRNA most likely to occur? □ (A) Absence of tRNA □ (B) Binding to miRNA □ (C) Methylation of DNA □ (D) Mutation of mRNA □ (E) Upregulation of mtDNA

Binding to miRNA (micro-RNAs are encoded by 5% of the human genome; do not encode for proteins; bind and inactivate or cleave mRNA, preventing translation of proteins by mRNA; silence gene expression without affecting the gene directly)

A 75-year-old woman has had a constant, dull pain in her back that has persisted for more than a month. She is in no acute distress. On physical examination, there are no abnormal findings. Laboratory findings include serum Na+, 141 mmol/L; K+, 4.4 mmol/L; Cl−, 103 mmol/L; CO2, 28 mmol/L; glucose, 101 mg/dL; creatinine, 0.9 mg/dL; urea nitrogen, 17 mg/dL; total protein, 6.8 g/dL; albumin, 4.2 g/dL; total bilirubin, 0.8 mg/dL; AST, 25 U/L; ALT, 29 U/L; calcium, 10.8 mg/dL; phosphorus, 2.3 mg/dL; and alkaline phosphatase, 228 U/L. Parathormone-related peptide is increased. What is the most likely diagnosis? □ (A) Bone metastases □ (B) Chondrosarcoma □ (C) Chronic cholecystitis □ (D) Osteoarthritis □ (E) Osteoporosis □ (F) Paget disease of bone

Bone metastases (prevalence of cancer increases with age; in absence of other findings, metastatic carcinoma involving bone should be suspected)

A 30-year-old man visits the physician because he has had joint pain in the right hip and left elbow and a headache for the past week. One month ago, he had similar pain in the left hip and knee, which slowly resolved. He remembers having a ringlike skin rash on his left thigh several months ago after a tick bite. On physical examination, there is joint tenderness, but no swelling or deformity of the right hip and left elbow. His heart rate is slightly irregular. What infectious agent is most likely to produce these findings? □ (A) Streptococcus pyogenes □ (B) Staphylococcus aureus □ (C) Borrelia burgdorferi □ (D) Mycobacterium tuberculosis □ (E) Parvovirus B19 □ (F) Yersinia enterocolitica

Borrelia burgdorferi (patient has Lyme arthritis, meningitis, and carditis and past history of erythema chronicum migrans; arthritis can appear weeks to a couple years after bite from deer tick)

A 45-year-old, previously healthy woman has had a chronic nonproductive cough for the past 2 months. During the past 3 weeks, she has had increasing dyspnea and arthralgias. One week ago, her cough was productive of blood-streaked sputum. She does not smoke. Physical examination shows temperature of 37.5°C, pulse of 77/min, respirations of 17/min, and blood pressure of 140/90 mm Hg. On auscultation, bilateral crackles are audible in the lungs. A chest radiograph shows bilateral nodular and cavitary infiltrates, but there are no masses. Laboratory findings show hemoglobin, 11.7 g/dL; hematocrit, 35.2%; platelet count, 217,000/mm3; WBC count, 6330/mm3; serum glucose, 72 mg/dL; creatinine, 2.6 mg/dL; and urea nitrogen, 25 mg/dL. Urinalysis shows specific gravity of 1.017, pH 6.5, 1+ proteinuria, 2+ hematuria, and no glucose or ketones. A transbronchial biopsy specimen shows necrotizing granulomatous vasculitis of the alveolar capillaries and small peripheral pulmonary arteries. A renal biopsy specimen shows a crescentic glomerulonephritis. Which of the following serologic test results is most likely to be positive? □ (A) Anti-DNA topoisomerase I □ (B) Anti-glomerular basement membrane antibody □ (C) Anti-Jo-1 antibody □ (D) Antimitochondrial antibody □ (E) C-ANCA □ (F) ANA □ (G) Antiribonucleoprotein antibody

C-ANCA (patient has Wegener's granulomatosis; multisystem vasculitis that most often involves the lungs and kidneys; C-ANCA is positive in 95% of Wegener's patients)

A 32-year-old woman has had increasing malaise and a 10-kg weight loss over the past 6 months. Physical examination shows muscle wasting, and there is a tan-yellow, plaquelike coating on her tongue. Laboratory testing shows hemoglobin, 12.6 g/dL; hematocrit, 37.8%; MCV, 85 μm3; platelet count, 188,300/mm3; and WBC count, 4320/mm3 with 73% segmented neutrophils, 3% bands, 9% lymphocytes, 14% monocytes, and 1% eosinophils. A scraping of the material from her tongue microscopically shows budding cells with pseudohyphae. She develops watery diarrhea 6 months later; a stool specimen contains cysts of Cryptosporidium parvum. She then develops a fever, cough, and severe dyspnea. Bronchoalveolar lavage is done; the figure shows the microscopic findings. Which of the following laboratory findings is most likely to be present? □ (A) ANA titer 1:1024 □ (B) CD4+ lymphocyte count 111/μL □ (C) Complement C2 undetectable □ (D) IgG 88 mg/dL □ (E) Neutrophil oxidative burst assay less than 5% □ (F) Positive RPR

CD4+ lymphocyte count 111/μL (patient has Pneumocystis jiroveci pneumonia; spectrum of opportunistic infections, wasting syndrome, and lymphopenia suggest AIDS complicating HIV infection)

A 10-year-old girl has a respiratory tract infection and is treated with trimethoprim-sulfamethoxazole. Three days later, she develops a sore throat, malaise, fever, and a macular skin rash on the trunk and extremities. Some of the skin lesions have a central raised area of more pronounced erythema. Within a few days, there are erosions of the oral mucosa and small blisters developing on purpuric skin macules. The blisters enlarge slightly and then show epidermal detachment. The total body surface area involved with blistering and detachment is less than 10%. This disease process is most likely mediated by what inflammatory cell type? □ (A) CD8+ lymphocytes □ (B) Eosinophils □ (C) Langerhans cells □ (D) Macrophages □ (E) Neutrophils □ (F) Natural killer cells

CD8+ lymphocytes (patient has Stevens-Johnson syndrome; severe form of erythema multiforme that can complicate infections and drug therapy; CD8+ T cells mediate SJS through epidermal cell necrosis)

A 30-year-old man is infertile and has a low sperm count. He also has chronic diarrhea with elevated quantitative stool fat. He has had recurrent, severe respiratory tract infections since early childhood. As a neonate, he had bowel obstruction from meconium ileus. He is most likely to have an abnormality involving which of the following genes? □ (A) CFTR □ (B) FGFR □ (C) G6PD □ (D) HFE □ (E) NF1 □ (F) p53

CFTR (patient has findings consistent with CF; agenesis of the vas deferens is common and leads to infertility)

An 8-year-old girl exhibits lethargy and somnolence with dizziness and weakness after a 6-hour bus ride returning from summer camp. On physical examination, her temperature is 37°C, pulse is 107/min, respiratory rate is 28/min, and blood pressure is 130/85 mm Hg. Breath sounds are audible in all lung fields with no wheezes or crackles. A chest x-ray shows no abnormalities. Arterial blood gas analysis shows pH 7.35, Po2 95 mm Hg, Pco2 37 mm Hg, and HCO3 − 20 mEq/L. Pulse oximetry shows an oxygen saturation of 90%, but the spectrophotometrically measured oxyhemoglobin saturation is 60%. Her blood lactic acid is 8 mmol/L, and total creatine kinase is 445 U/L with MB fraction of 5%. She is given 100% oxygen in a hyperbaric chamber and improves in 20 minutes. She is most likely to have experienced poisoning with which of the following? □ (A) Aspirin (acetylsalicylic acid) □ (B) Carbon monoxide □ (C) Iron sulfate □ (D) Lead □ (E) Methanol □ (F) Organophosphate insecticide

Carbon monoxide (binds to Hb 200x more avidly than oxygen, which leads to reduced oxygen saturation; CO binds cardiac myoglobin even more avidly than Hb; tissue hypoxia leads to lactic acidosis and cardiac and skeletal muscle begin to break down, releasing creatine kinase)

For the past month, a 16-year-old girl has had increasing heat intolerance, with nervousness and a fine tremor. She has had irregular menstrual cycles since menarche 2 years ago and has not menstruated for 3 months. She has not used contraceptives. One week ago, she noticed a small amount of vaginal bleeding and now has sudden onset of severe abdominal pain. On physical examination, her temperature is 36.9°C, pulse is 105/min, respirations are 17/min, and blood pressure is 80/40 mm Hg. There is marked right upper quadrant abdominal tenderness, and bowel sounds are reduced. A stool sample is negative for occult blood. Brownish fluid is noted in the vaginal vault, emanating from a reddish brown, 2-cm mass in the vault. An abdominal ultrasound examination shows several 3- to 6-cm masses in the liver, and the uterus appears enlarged, with a 5-cm mass. An anteroposterior chest radiograph shows several 1- to 3-cm nodules in the lungs. Paracentesis is done, and there is blood in the aspirate. The figure shows the microscopic appearance of a biopsy specimen of the vaginal mass. She is given methotrexate and actinomycin D and improves. What neoplasm would most likely produce these findings? □ (A) Adenocarcinoma □ (B) Choriocarcinoma □ (C) Clear cell carcinoma □ (D) Leiomyosarcoma □ (E) Malignant mixed müllerian tumor □ (F) Sarcoma botryoides

Choriocarcinoma (most aggressive and least common form of molar pregnancy; can metastasize widely, particularly to lung, vagina, brain, liver, and kidney; neoplasm has malignant-appearing syncytiotrophoblast and forms of soft, hemorrhagic mass that can rupture and bleed)

A 62-year-old woman who has had a chronic cough for years becomes short of breath after climbing a single flight of stairs. A chest radiograph performed 1 year ago showed increased lucency of upper lung fields and bilateral flattening of the diaphragmatic leaves. She has had nausea and vague abdominal discomfort for 6 months. Biopsy specimens from an upper gastrointestinal endoscopic study show a chronic nonspecific gastritis with no detectable Helicobacter pylori organisms. During the past month, she has passed red-colored urine on several occasions. Cystoscopic examination shows a 3-cm exophytic mass in the dome of the bladder, and biopsy specimens show a transitional cell carcinoma. What is the most likely risk factor for this spectrum of findings? □ (A) α1-Antitrypsin deficiency □ (B) Chronic alcoholism □ (C) Cigarette smoking □ (D) Exposure to aniline dye □ (E) Vitamin C deficiency

Cigarette smoking (patient has findings that are consistent with emphysema; smoking promotes gastritis)

A 25-year-old man is involved in a rollover accident in which he is ejected from the vehicle. He sustains a compound fracture of the right humerus and undergoes open reduction with internal fixation of the humeral fracture. Three days later, he has marked swelling of the right arm and palpable crepitus. A Gram stain of exudate from the wound site has the appearance shown in the figure. What organism is the most likely causative agent for this patient's infection? □ (A) Candida albicans □ (B) Listeria monocytogenes □ (C) Haemophilus influenzae □ (D) Clostridium perfringens □ (E) Bacteroides fragilis

Clostridium perfringens (large, gram positive rods seen in the figure are characteristic of Clostridium perfringens; can contaminate open wounds and produce gas gangrene)

A 19-year-old man is found unconscious by his roommate and taken to the emergency department. On physical examination, temperature is 41.2°C, pulse is 103/min, respirations are 18/min and shallow, and blood pressure is 145/100 mm Hg. He develops an intractable cardiac dysrhythmia and dies. At autopsy, the heart is slightly enlarged; microscopically, the distal coronary arteries are thickened. Sections of the brain show a 2-cm area of hemorrhage in the right superior parietal lobe and a 0.5-cm hemorrhage in the medulla. There is a partially cystic, 1-cm area with brown discoloration in the left anterior frontal lobe. What drug was this patient most likely using regularly? □ (A) Amphetamine □ (B) Barbiturate □ (C) Cocaine □ (D) Ethanol □ (E) Heroin □ (F) Marijuana □ (G) Methamphetamine □ (H) Phencyclidine

Cocaine (man has features of acute cocaine toxicity with hyperthermia and chronic cocaine use with coronary arteriopathy and cerebral hemorrhagic strokes; cocaine is a powerful vasoconstrictor)

A 25-year-old man has had increasingly frequent infections over the past 5 years. His most recent respiratory infection was due to Streptococcus pneumoniae. He now has watery diarrhea. On physical examination, he is below ideal weight. There is a vesicular rash in the T10 dermatomal distribution on the left. Laboratory findings include hemoglobin, 14.3 g/dL; hematocrit, 43.2%; platelet count, 290,600/mm3; and WBC count, 7200/mm3 with 55% segmented neutrophils, 2% bands, 35% lymphocytes, 6% monocytes, and 2% eosinophils. Quantitative immunoglobulins include IgA of 22 mg/dL, IgG of 175 mg/dL, and IgM of 40 mg/dL. Lymphocyte subsets by flow cytometry show CD4+ cells (absolute) of 630/μL, CD8+ cells (absolute) of 785/μL, B cells of 280/μL, and T cells of 2010/μL. A stool culture is negative for bacterial pathogens, but a stool culture for ova and parasites shows Giardia lamblia cysts. What is the most likely diagnosis? □ (A) Chronic granulomatous disease □ (B) Common variable immunodeficiency □ (C) Hyper-IgM syndrome □ (D) Leukocyte adhesion deficiency □ (E) Severe combined immunodeficiency

Common variable immunodeficiency (normal numbers of T cells with normal to low numbers of B cells; hypogammaglobulinemia with decreased IgG and possibly other immunoglobulin types; occurs in young adults of both sexes; increased bacterial infection and giardiasis and recurrent herpes simplex)

A 68-year-old man has had increasing dyspnea with cough productive of frothy sputum for the past 5 months. On physical examination, he is afebrile, and his blood pressure is 165/100 mm Hg. There is dullness to percussion at lung bases. He has pitting edema of the ankles. A chest radiograph shows blunting of costophrenic recesses bilaterally and cardiomegaly with prominent right and left heart borders. A right thoracentesis is performed, and 300 mL of straw-colored fluid is removed. Laboratory studies on this fluid show total protein of 2.2 g/dL (serum is 6.5 g/dL), glucose of 45 mg/dL (serum is 75 mg/dL), lactate dehydrogenase of 200 U/L (serum is 420 U/L), pH 7.2, and cell count of 100/mm3 mononuclear leukocytes, and no red blood cells. What condition does he most likely have? □ (A) Cavitary tuberculosis □ (B) Congestive heart failure □ (C) Malignant mesothelioma □ (D) Non-Hodgkin lymphoma □ (E) Pneumococcal pneumonia □ (F) Small-cell anaplastic carcinoma

Congestive heart failure (pleural fluid findings with low protein, low LDH, and low cell count are consistent with a transudate with hydrothorax; HTN has probably led to left heart failure with pulmonary edema; long standing left heart failure can lead to right heart failure with body cavity effusions and peripheral edema)

A 22-year-old woman has experienced bizarre changes in her behavior over the past year, culminating in her withdrawal from college. She is diagnosed with bipolar disorder. Over the next year, she develops neurologic manifestations that include resting and intention tremors, rigidity, chorea, dysphagia, and dysarthria. On physical examination, she has bilateral Babinski responses. There are ringlike deposits of gold-colored material involving the cornea bilaterally, but her vision is not decreased. One year later, she has an illness that lasts several weeks, with nausea, vomiting, and malaise and scleral icterus. Laboratory findings include serum AST, 100 U/L; ALT, 122 U/L; alkaline phosphatase, 105 U/L; total bilirubin, 4.5 mg/dL; glucose, 77 mg/dL; and creatinine, 0.9 mg/dL. Serologic test results for hepatitis A, B, and C are negative. This episode subsides without treatment, but she eventually develops cirrhosis. A mutation in a gene encoding for what substance is most likely to be present? □ (A) α1-Antitrypsin □ (B) CFTR □ (C) Galactose-1-phosphate uridyl transferase □ (D) Glucocerebrosidase □ (E) Glucose-6-phosphatase □ (F) Copper-transporting ATPase

Copper-transporting ATPase (patient has Wilson's disease with Kayser-Fleischer corneal rings; mutation in gene encoding for copper-transporting ATPase; excessive copper deposition in liver, putamen, and cornea)

A 39-year-old man sees the physician because he has had a severe headache for 4 days. On physical examination, he has generalized muscle wasting. On funduscopic examination, there is no papilledema, but changes are present that suggest cytomegalovirus retinitis. A lumbar puncture is performed, and there is a slightly increased opening pressure at 200 mm H2O. Laboratory analysis of the CSF shows 22/mm3 mononuclear leukocytes, no neutrophils, no RBCs, glucose 60 mg/dL (serum 85 mg/dL), and protein 59 mg/dL. An India ink preparation of the CSF is shown in the figure. His CD4+ lymphocyte count is 106/mm3. Which of the following organisms is most likely to produce these findings? □ (A) Staphylococcus aureus □ (B) Mycobacterium tuberculosis □ (C) Listeria monocytogenes □ (D) Toxoplasma gondii □ (E) Cryptococcus neoformans

Cryptococcus neoformans (patient's immunocompromised state consistent with AIDS from HIV infection has resulted in infection with Cryptococcus neoformans; infection probably starts in lungs, but CNS is second most commonly involved area; meningitis is the most typical manifestation)

A 72-year-old woman with chronic bronchitis from cigarette smoking has been bedridden because of influenza virus infection for the past 2 weeks. She experiences sudden, severe dyspnea with chest pain. On examination, her temperature is 37°C, pulse is 104/min, respiratory rate is 26/min, and blood pressure is 100/60 mm Hg. Her left leg is swollen and painful on raising. She appears cyanotic. A systolic ejection sound and diastolic murmur are auscultated over the pulmonic region. A chest x-ray shows a prominent right heart border. Echocardiography shows blood flow from the right to left atrium. Laboratory studies show an elevated D-dimer. She then develops right leg weakness. MRI of the brain shows early infarction within the left hemisphere. Which of the following cardiac conditions best explains these findings? □ (A) Constrictive pericarditis □ (B) Dilated cardiomyopathy □ (C) Eisenmenger complex □ (D) Infective endocarditis □ (E) Rheumatic heart disease

Eisenmenger complex (paradoxical embolus explains her findings; patient has thrombophlebitis with pulmonary embolism as a rest of prolonged bed rest owing to her viral pneumonia; COPD led to cor pulmonale, reversing the shunt across the atrial septal defect and allowing a thromboembolus arising the venous circulation to reach the systemic arterial circulation in the brain)

A 68-year-old man has had headaches with increasing fatigue and dyspnea for the past 2 years. On physical examination, there are crackles at the lung bases. Funduscopy shows retinal arteries with a "silver wire" appearance. The figure shows the representative appearance of his brain. A chest x-ray shows a prominent left heart border. He then has sudden loss of consciousness, and head CT scan now shows a right putamen hematoma. Laboratory studies show serum glucose of 77 mg/dL, creatinine of 2.5 mg/dL, and total cholesterol of 181 mg/dL. What underlying conditions does he most likely have? □ (A) Chronic alcohol abuse □ (B) Diabetes mellitus □ (C) Essential hypertension □ (D) Rheumatic heart disease □ (E) Sickle cell anemia □ (F) Systemic lupus erythematosus

Essential hypertension (findings point to prolonged and sustained systemic hypertension that has led to left ventricular hypertrophy, hypertensive retinopathy, and chronic renal failure with arteriosclerosis)

A 44-year-old woman has noted dull right upper quadrant pain for the past year. On physical examination, there is right upper quadrant tenderness on palpation. Abdominal CT scan shows a 5-cm, circumscribed mass in the superior right lobe of the liver. The figure shows the representative gross appearance of a similar mass. She experiences sudden onset of dyspnea with diaphoresis 1 month later. Multiple peripheral perfusion defects are seen on a pulmonary ventilation/perfusion scan. Which of the following combinations of pharmacologic agents taken by this patient regularly is most likely to be associated with these findings? □ (A) Allopurinol and sulfamethoxazole □ (B) Ethynyl estradiol and norethindrone □ (C) Ibuprofen and acetylsalicylic acid □ (D) Isoniazid and rifampicin □ (E) Phenacetin and acetaminophen □ (F) Prednisone and cyclophosphamide

Ethynyl estradiol and norethindrone (thromboembolic events and hepatic adenoma are rare complications of oral contraceptive use)

A 22-year-old woman incurs multiple blunt trauma with bilateral femoral and right humeral fractures from a fall. The fractures are treated with open reduction and internal fixation. She is in stable condition until 3 days later, when she becomes progressively delirious and then comatose. On physical examination, she is afebrile. Head CT scan shows generalized brain edema. The representative gross appearance of her brain is shown in the figure. Lumbar puncture yields clear CSF with no RBCs, one mononuclear cell, and normal protein and glucose. Her serum glucose is 102 mg/dL, and creatinine is 0.9 mg/dL. What is the most likely diagnosis? □ (A) Abscess with Staphylococcus aureus □ (B) Central pontine myelinolysis □ (C) Diffuse axonal injury □ (D) Fat embolism □ (E) Ruptured berry aneurysm □ (F) Viral meningitis

Fat embolism (delay in onset after trauma is consistent with fat embolism to the brain; figure shows the predominantly white matter petechiae of brain purpura)

A 17-year-old girl has had frequent nosebleeds since childhood. Her gums bleed easily, even with routine tooth brushing. She has experienced menorrhagia since menarche at age 14. On physical examination, there are no abnormal findings. Laboratory studies show hemoglobin, 14.1 g/dL; hematocrit, 42.5%; MCV, 90 μm3; platelet count, 277,400/mm3; and WBC count, 5920/mm3. Her platelets fail to aggregate in response to ADP, collagen, epinephrine, and thrombin. The ristocetin agglutination test result is normal. There is a deficiency of glycoprotein IIb/IIIa. Prothrombin time is 12 seconds, and partial thromboplastin time is 28 seconds. What is the most likely diagnosis? □ (A) Disseminated intravascular coagulation □ (B) Glanzmann thrombasthenia □ (C) Immune thrombocytopenic purpura □ (D) Vitamin C deficiency □ (E) Von Willebrand disease

Glanzmann thrombasthenia (rare autosomal recessive disorder; defective platelet aggregation from deficiency or dysfunction of glycoprotein IIb/IIIa)

A 39-year-old man states that he has had watery diarrhea and flatulence for the past 8 months. He also reports increasing fatigue and a 4-kg weight loss. He has had urticarial plaques on extensor surfaces of the elbows and knees and on the upper back for the past month. Some of the plaques have small, grouped vesicles. A biopsy specimen of one of the skin lesions shows neutrophils at the tips of dermal papillae with overlying basal cell vacuolization. Under immunofluorescence microscopy, granular IgA deposits appear at the tips of dermal papillae. The figure shows the microscopic appearance of a jejunal biopsy specimen. More than 20 years later, the patient has a T-cell lymphoma of the jejunum. Antibody to which of the following is most likely to produce this disease? □ (A) Desmoglein 3 □ (B) Double-stranded DNA □ (C) Gliadin □ (D) Histone □ (E) Ribonucleoprotein □ (F) Type IV collagen

Gliadin (man has celiac disease complicated by dermatitis herpetiformis; jejunal biopsy specimen shows mucosal flattening and increased intraepithelial lymphocytes; predisposed if you are HLA-DQ2 or HLA-DQ8)

A 10-month-old infant is not meeting developmental milestones and is below ideal weight and height. The parents noted an episode of convulsions 1 week ago. On physical examination, the infant has hepatosplenomegaly and generalized non-tender lymphadenopathy. There is tenderness on palpation of the right upper extremity. No focal neurologic deficits are present, but the infant's attention and movement are diminished. A radiograph of the right arm shows a healing fracture. Laboratory findings show hemoglobin of 9.7 g/dL, hematocrit of 28.4%, platelet count of 76,700/mm3, WBC count of 4200/mm3, glucose of 78 mg/dL, and creatinine of 0.4 mg/dL. A bone marrow biopsy is done; the figure shows the microscopic appearance of the biopsy specimen. The infant is most likely to have the near absence of which of the following enzymes? □ (A) α-l-Iduronidase □ (B) α-1,4-Glucosidase □ (C) Arylsulfatase A □ (D) Glucocerebrosidase □ (E) Glucose-6-phosphatase □ (F) Hexosaminidase A □ (G) Hexosaminidase B □ (H) Sphingomyelinase

Glucocerebrosidase (infant has acute neuronopathic form of Gaucher disease type II; uniformly fatal in children; marrow is infiltrated by large cells with abundant pale cytoplasm with the appearance of crumpled tissue paper)

A 35-year-old man has had bouts of severe, diffuse abdominal pain over the past 4 months. These bouts have been accompanied by fever, malaise, and myalgias. During one bout, he sees the physician. On physical examination, his temperature is 37.7°C, pulse is 81/min, respirations are 16/min, and blood pressure is 145/90 mm Hg. There is diffuse abdominal tenderness, but no masses, and bowel sounds are present. A stool sample is positive for occult blood. Laboratory findings show serum glucose of 73 mg/dL, amylase of 44 U/L, AST of 54 U/L, ALT of 23 U/L, creatinine of 2.4 mg/dL, and urea nitrogen of 22 mg/dL. A renal biopsy specimen shows acute transmural vasculitis of medium-sized arteries; the glomeruli and tubules are unremarkable. Mesenteric artery angiography is performed; the figure shows the appearance of the angiogram. Which of the following serologic tests is most likely to be positive in this patient? □ (A) Antimitochondrial antibody □ (B) C-ANCA □ (C) ANA □ (D) Cryptococcus neoformans antigen □ (E) HBsAg □ (F) Histoplasma capsulatum antibody □ (G) p24 antigen

HBsAg (patient has findings of classic polyarteritis nodosa, and in 1/3 cases the HBsAg is positive)

A 37-year-old primigravida at 30 weeks' gestation has noted increasing pedal edema for the past 2 weeks. During the past week, she has developed headaches and confusion, and she has decreased urine output. She exhibited seizure activity and then lapsed into a coma. On physical examination, her temperature is 36.8°C, pulse is 82/min, respirations are 18/min, and blood pressure is 145/95 mm Hg. Her heart rate is regular, and lung fields are clear. The abdomen is soft, and bowel sounds are present. There is pitting edema to the thighs. No vaginal bleeding is noted, and the cervix is not effaced. Laboratory findings show hemoglobin, 11.9 g/dL; hematocrit, 35.8%; platelet count, 73,000/mm3; WBC count, 8180/mm3; glucose, 151 mg/dL; total protein, 6.6 g/dL; albumin, 3.2 g/dL; total bilirubin, 2.3 mg/dL; AST, 78 U/L; ALT, 93 U/L; alkaline phosphatase, 253 U/L; and prothrombin time, 32 seconds. Urinalysis shows specific gravity of 1.024, pH 6, 4+ proteinuria, 1+ glucosuria, and no blood. An ultrasound examination shows a viable 30-week fetus. What condition is most likely present in this patient? □ (A) Abruptio placentae □ (B) Budd-Chiari syndrome □ (C) Dilated cardiomyopathy □ (D) HELLP syndrome □ (E) Hydatidiform mole □ (F) Reye syndrome □ (G) Sheehan syndrome

HELLP syndrome (risk factors: nulliparity, advanced maternal age, DM, hypertension, prior history of preeclampsia, and renal disease; patients may progress to DIC; emergent delivery is indicated)

A 40-year-old man has been bothered by oral candidiasis for the past year. On physical examination, he has muscle wasting. His weight is 70% of normal for his height and age. He has generalized nontender lymphadenopathy, but no hepatosplenomegaly. The figure shows a skin lesion on his forearm. Laboratory findings show hemoglobin, 12.2 g/dL; hematocrit, 36.5%; MCV, 85 μm3; platelet count, 188,000/mm3; and WBC count, 3460/mm3 with 78% segmented neutrophils, 4% bands, 10% lymphocytes, 6% monocytes, and 2% eosinophils. Infection with what organism is most likely to produce these findings? □ (A) Hepatitis C virus □ (B) Herpes simplex virus □ (C) HIV □ (D) Mycobacterium leprae □ (E) Plasmodium vivax □ (F) Staphylococcus aureus □ (G) Streptococcus pyogenes

HIV (reddish purple nodular lesion is typical of Kaposi sarcoma in a patient with wasting syndrome, oral thrush, and lymphopenia characteristic of HIV infection with AIDS)

In a study of immunodeficient patients, samples of patient blood are collected and analyzed by flow cytometry. The figure at the bottom of the page shows the results for one group of patients. These patients are statistically more likely to develop non-Hodgkin lymphomas. Which diagnosis is most applicable to this group of patients? □ (A) Bruton agammaglobulinemia □ (B) Common variable immunodeficiency □ (C) DiGeorge anomaly □ (D) HIV infection □ (E) Hyper-IgM syndrome □ (F) Postchemotherapy state □ (G) Severe combined immunodeficiency

HIV infection (patients have decreased CD4+ count compared to CD8+; HIV selectively infects CD4+ cells and leads to reduced CD4+ which presages AIDS)

A 29-year-old man sees his physician because of burning pain on urination that has persisted for 3 days. There is a urethral discharge. A sample of the exudate is positive by ELISA for Chlamydia trachomatis. The man has increasing stiffness of the knees and ankles and lower back pain 3 weeks later. A radiograph of the lumbar spine shows narrowing with sclerosis of the sacroiliac joints. One month later, he develops painful erythema of the penile glans, and the conjunctivae are red. A follow-up examination shows a slightly irregular heart rate and a murmur suggestive of aortic regurgitation. The back pain continues off and on for 5 more months. Which of the following test results is most likely to be positive? □ (A) ANCA □ (B) ANA □ (C) HLA-B27 antigen □ (D) Lyme disease □ (E) Purified protein derivative □ (F) Rapid plasma reagin □ (G) Rheumatoid factor □ (H) U1-RNP

HLA-B27 antigen (combination of nongonococcal urethritis, arthritis, and conjunctivitis suggests Reiter syndrome, one of the spondyloarthropathies)

A 20-year-old man has sudden onset of severe abdominal and back pain and dyspnea. His medical history indicates similar episodes over a 12-year period. He had osteomyelitis of the left hip 1 year ago; the bone culture was positive for Salmonella enteritidis. On physical examination, his lungs are clear to auscultation, but he has tachycardia. Palpation of the abdomen reveals diffuse tenderness with rigidity of abdominal musculature, but no apparent masses. CT scan of the chest shows prominent pulmonary veins, but no infiltrates. Abdominal CT scan shows the presence of multiple 0.5- to 1-cm stones in the gallbladder, a very small spleen, and prominent hepatic veins. An abdominal radiograph shows no free air. CBC shows hemoglobin of 10.2 g/dL, hematocrit of 30.9%, MCV of 99 μm3, RDW of 22, platelet count of 189,300/mm3, and WBC count of 6320/mm3. What additional laboratory test finding is most likely to be reported? □ (A) Amylase 694 U/L □ (B) Positive anticardiolipin antibody □ (C) Calcium 12.3 mg/dL □ (D) Ferritin 710 ng/mL □ (E) Haptoglobin 1 mg/dL □ (F) Triglyceride 1140 mg/dL

Haptoglobin 1 mg/dL (patient has findings that are consistent with sickle cell anemia with abdominal crisis and vertebral bone marrow infarction; haptoglobin is low as a result of sickle cell crisis with hemolysis)

A 40-year-old man has been taking daily insulin injections for the past 25 years. When he does not arrive at work, a friend visits his house and finds him on the floor in an obtunded state. He is taken to the hospital by ambulance. On admission to the hospital, he cannot be aroused. He is afebrile, with a pulse of 90/min, respirations of 17/min, and blood pressure of 90/60 mm Hg. Laboratory studies show a hemoglobin A1c concentration of 8.9%, serum glucose level of 11 mg/dL, and serum osmolality of 295 mOsm/kg. Urinalysis shows 4+ ketonuria with a specific gravity of 1.010. Which of the following statements best characterizes these findings? □ (A) He is in poor glycemic control, and has had an insulin overdose □ (B) He is in good glycemic control, but has developed ketoacidosis □ (C) He is in poor glycemic control, and is not taking his insulin □ (D) He is in good glycemic control, but has not eaten food recently □ (E) He is in poor glycemic control, and has developed a hyperosmolar coma

He is in poor glycemic control, and has had an insulin overdose (increased HbA1c suggests patient has poorly controlled hyperglycemia; profound hypoglycemia is consistent with insulin overdose)

A 59-year-old man comes to the physician seeking a prescription for sildenafil after seeing television advertisements regarding erectile dysfunction. He is 168 cm (5 ft 6 in) tall and weighs 93 kg (body mass index 33). On physical examination, there are bilateral carotid bruits and a midline palpable abdominal pulsatile mass. Decreased hair is noted over the lower extremities, and a 1-cm shallow ulceration is present in the skin over the right first metatarsal head. He has decreased sensation to light touch and pinprick in the lower extremities. Laboratory findings include hemoglobin, 12.9 g/dL; hematocrit, 42%; WBC count, 8950/mm3; Na+, 140 mmol/L; K+, 4.2 mmol/L; Cl−, 105 mmol/L; CO2, 26 mmol/L; glucose, 144 mg/dL; and creatinine, 1.7 mg/dL. Which of the following laboratory findings is most likely to be present in this man? □ (A) Plasma ACTH 119 pg/mL □ (B) Hemoglobin A1c 8.8% □ (C) Plasma homocysteine 23 μmol/L □ (D) Cerebrospinal fluid oligoclonal IgG bands □ (E) Serum anti-parietal cell antibodies

Hemoglobin A1c 8.8% (Hb A1c elevation occurs in DM; patient has complications of DM type 2 that are typical of obesity)

A 30-year-old woman has noted a 5-kg weight gain over the past 3 months; she has not had a menstrual period during that time. She has experienced upper abdominal pain for the past month. Physical examination shows abdominal enlargement with apparent ascites. There is no peripheral edema. She has a positive pregnancy test. Additional laboratory findings show hemoglobin, 13.2 g/dL; hematocrit, 39.7%; WBC count, 12,300/mm3; glucose, 80 mg/dL; AST, 581 U/L; ALT, 611 U/L; total bilirubin, 1.3 mg/dL; total protein, 6.2 g/dL; and albumin, 3.5 g/dL. An abdominal ultrasound scan shows hepatomegaly with heterogeneous echogenicity, and there is an intrauterine gestation with a fetus estimated at 12 weeks' size. What pathologic finding is most likely to be present? □ (A) Choledocholithiasis □ (B) Chronic passive congestion □ (C) Hepatic venous thrombosis □ (D) Hepatocellular adenoma □ (E) Metastatic choriocarcinoma □ (F) Microvesicular steatosis

Hepatic venous thrombosis (patient has Budd-Chiari syndrome; can complicate pregnancy or the postpartum state; hepatic venous occlusion leads to hepatomegaly with severe centrilobular congestion and necrosis; more pronounced than nutmeg liver of chronic passive congestion with right-sided heart failure)

A 43-year-old woman has had easy fatigability along with pain and stiffness of both wrist joints and other small joints of the hands for the past 7 years. The stiffness is marked in the morning and abates as the day goes by. X-rays of her hands reveal narrowing of the proximal interphalangeal and metacarpophalangeal joint spaces from synovitis and erosion of the cartilage. Laboratory tests show Hgb 8.4 g/dL, Hct 23.5, MCV 65 fL, and MCH 23 pg. Her peripheral blood smear shows hypochromic, microcytic red blood cells. Her serum iron and iron binding capacity levels are low and the ferritin level elevated. A high level of which of the following is most likely related to the causation of anemia in this woman? □ (A) C-reactive protein □ (B) GM-CSF □ (C) Hepcidin □ (D) IgG □ (E) IL-1β

Hepcidin (patient has rheumatoid arthritis with anemia of chronic disease; despite abundant stored iron such anemias are caused by impaired transfer of iron from macrophages to developing erythroid cells)

A 47-year-old white man has had increasing orthopnea and worsening pedal edema for the past 3 years. He complains of worsening arthritis involving his hands, knees, hips, and elbows. On physical examination, he has decreased range of motion of the lower legs, but no apparent joint deformities, warmth, or swelling. There is a brownish hue to his skin, although it is winter, and he rarely goes outdoors. Laboratory findings show hemoglobin, 13.7 g/dL; hematocrit, 40.8%; MCV, 90 μm3; platelet count, 213,500/mm3; WBC count, 6690/mm3; Na+, 141 mmol/L; K+, 4.2 mmol/L; Cl−, 101 mmol/L; CO2, 24 mmol/L; glucose, 201 mg/dL; creatinine, 1.2 mg/dL; and calcium, 8.2 mg/dL. Which of the following underlying diseases most likely explains these findings? □ (A) β-Thalassemia minor □ (B) Diabetes mellitus type 1 □ (C) Familial hypercholesterolemia □ (D) Hereditary hemochromatosis □ (E) Rheumatoid arthritis

Hereditary hemochromatosis (C282Y mutation in HFE gene can be found in people with increased absorption of dietary iron; leads to organ dysfunction involving the heart, pancreas, skin, and joints in men)

A 22-year-old woman has sudden onset of severe lower abdominal pain. Her medical history is remarkable for Chlamydia trachomatis cervicitis. On physical examination, her temperature is 36.9°C, pulse is 90/min, respirations are 17/min, and blood pressure is 90/50 mm Hg. There is lower abdominal tenderness, but no palpable masses. No vaginal bleeding is present. The rectal examination is unremarkable, and a stool sample is negative for occult blood. Bowel sounds are reduced. An abdominal ultrasound scan is performed, and the uterus appears normal in size with no masses visualized, but there is a right adnexal mass. Culdocentesis is performed, and there is blood in the aspirate. Laboratory findings show hemoglobin of 9.5 g/dL, hematocrit of 28.6%, platelet count of 269,300/mm3, and WBC count of 9110/mm3. Which of the following laboratory findings is most likely to be present? □ (A) Carcinoembryonic antigen increase □ (B) Entamoeba histolytica cysts in stool □ (C) Factor XIII deficiency □ (D) Follicle-stimulating hormone decrease □ (E) Human chorionic gonadotropin elevation □ (F) Partial thromboplastin time prolonged □ (G) Schistosoma haematobium eggs in urine

Human chorionic gonadotropin elevation (patient has findings suggestive of ruptured ectopic pregnancy; hCG would be elevated; gonococcal and chlamydial infection are risk factors for PID which increases the risk of ectopic pregnancy)

A 9-year-old girl has become increasingly listless over the past year. On physical examination, she has pitting edema to the thighs, muscle wasting, a protuberant abdomen with a fluid wave, areas of scaling skin with decreased pigmentation, and patches of hair that are irregularly pigmented. No ecchymoses are noted. She is 75% of ideal body weight, but her height is normal. Which of the following laboratory findings is most likely to be present? □ (A) Hyperhomocysteinemia □ (B) Hyperuricemia □ (C) Hypercalcemia □ (D) Hypoprothrombinemia □ (E) Hypoalbuminemia □ (F) Hypoinsulinemia

Hypoalbuminemia (child has protein-energy malnutrition characteristic of kwashiorkor; decreased protein intake translates to diminished albumin synthesis by the liver; decreased plasma oncotic pressure leads to peripheral edema and ascites; associated with hair and skin changes and muscle wasting)

A 90-year-old woman died suddenly. Autopsy shows a 5-cm skin ulceration extending to the sacrum. She had diffuse muscle wasting; a microscopic section shows decreased size of muscle fibers without inflammation or fibrosis. Her bones show marked osteoporosis, and there is vertebral column kyphosis. A finding on examination of the lungs is shown in the figure. There is a right lower lobe pneumonia. Which of the following conditions most likely predisposed this patient to the pathologic findings seen at autopsy? □ (A) Antiphospholipid syndrome □ (B) Aplastic anemia □ (C) Chronic alcoholism □ (D) Elder abuse with blunt trauma □ (E) Immobilization □ (F) Malnutrition

Immobilization (woman has features seen in chronically bedridden individuals; large saddle thromboembolus in the pulmonary arterial trunk)

A 31-year-old, G3, P2 woman is in the second trimester. Her prior pregnancies ended with delivery of normal term infants who are still living. She has an ultrasound examination because of lack of fetal movement by 18 weeks. The fetal ultrasound examination shows microcephaly with periventricular leukomalacia and calcifications. Her HIV test is positive; her serologic test for syphilis is negative. TORCH titers are performed on maternal blood: CMV: IgG 1.2 (normal <0.9); IgM 0.7 (normal <0.9); HSV1: IgG 1.2 (normal <0.9); IgM 0.6 (normal <0.9); HSV2: IgG 0.6 (normal <0.9); IgM 0.5 (normal <0.9); Rubella: IgG 15 (normal <10); IgM 0.4 (normal <0.9); Toxoplasma: IgG 11 (normal <9); IgM 3.3 (normal <0.9) Which of the following is the most likely risk factor for this fetal infection? □ (A) Ingestion of contaminated meat □ (B) Inhalation of droplet nuclei □ (C) Injection drug use □ (D) Mosquito bite □ (E) Previous blood transfusion □ (F) Sexual intercourse

Ingestion of contaminated meat (the only increased IgM titer, indicating recent infection, is that for Toxoplasma; fetal CNS findings are consistent with congenital toxoplasmosis; increased IgG suggests past infection)

A 33-year-old man has experienced onset of chest pain, diaphoresis, and dyspnea over the past 6 hours. In the emergency department, he has a serum troponin I level of 6 ng/mL. Additional laboratory findings include hematocrit of 41%, hemoglobin A1c of 4.2%, total serum cholesterol of 482 mg/dL, and serum triglyceride of 160 mg/dL. Emergent coronary angiography shows 65% stenosis of the left circumflex artery and 70% stenosis of the left anterior descending artery. Angioplasty is performed. He experiences a series of transient ischemic attacks 1 year later. He also has pain in the lower extremities when walking more than 300 m. He is given a drug that inhibits hepatic HMG CoA reductase. The pathogenesis of his underlying disease is most likely related to a reduction in which of the following? □ (A) β cells □ (B) Glucose transport proteins □ (C) Hepatocytes □ (D) Intimal smooth muscle cells □ (E) LDL receptors

LDL receptors (patient with acute MI, cerebrovascular disease, and peripheral vascular disease has features suggestive of heterozygous familial hypercholesterolemia; mutation in gene encoding for LDL receptor is present)

Several children 6 to 10 years old in the same community have been reported by the local physician to have similar symptoms. They all are doing poorly in school, which has been attributed to behavioral problems. Their parents state that these children have poor appetites, complain of nausea, and have frequent headaches. On physical examination, they have decreased sensation to touch over the lower extremities. They exhibit loss of fine motor control of movement and have a slightly ataxic gait. A representative CBC shows hemoglobin of 11.8 g/dL, hematocrit of 35.2%, MCV of 82 μm3, platelet count of 282,300/mm3, and WBC count of 4745/mm3. Examination of the peripheral blood smear shows basophilic stippling of the RBCs. Serum chemistries show Na+, 144 mmol/L; K+, 4.4 mmol/L; Cl−, 105 mmol/L; CO2, 26 mmol/L; glucose, 69 mmol/L; creatinine, 1.4 mg/dL; calcium, 7.7 mg/dL; total protein, 6.6 g/dL; albumin, 4.3 g/dL; AST, 43 U/L; ALT, 28 U/L; alkaline phosphatase, 189 U/L; and total bilirubin, 1.1 mg/dL. Excessive chronic ingestion of which of the following substances is most likely to explain these findings? □ (A) Fluoride □ (B) Lead □ (C) Methanol □ (D) Monosodium glutamate □ (E) Vitamin A □ (F) Zinc

Lead (findings are characteristic of lead poisoning which is mainly manifested by neurologic disorders; lead absorption is enhanced by zinc deficiency; lead inhibits heme incorporation into Hb, leading to increased amts of zinc protoporphyrin with anemia)

For the past month, a 33-year-old woman has had burning epigastric pain and nausea and vomiting. An upper gastrointestinal endoscopic study shows multiple 1-cm shallow gastric antral and proximal duodenal ulcerations. She is treated with omeprazole and improves. One year later, she has an episode of severe, colicky lower abdominal pain and hematuria and passes a calcium oxalate calculus. She notes galactorrhea 1 month later, and over the next 2 months ceases to menstruate. She is given a dopamine agonist and improves. Laboratory findings show Na+, 140 mmol/L; K+, 4 mmol/L; Cl−, 101 mmol/L; CO2, 25 mmol/L; calcium, 11.1 mg/dL; phosphorus, 2.4 mg/dL; and creatinine, 1.1 mg/dL. Which of the following gene mutations with associated neoplasms is most likely to develop in this patient? □ (A) MEN1—islet cell adenoma □ (B) RET—medullary carcinoma □ (C) BCL6—non-Hodgkin lymphoma □ (D) APC—osteoma □ (E) RET—pheochromocytoma □ (F) VHL—renal cell carcinoma

MEN1—islet cell adenoma (patient has MEN1 with gastrinoma, parathyroid lesion, and prolactinoma)

Over the past 6 months, a 42-year-old man has experienced increasing fatigue and a 6-kg weight gain, predominantly in a truncal distribution. On physical examination, his temperature is 37.1°C, pulse is 77/min, respirations are 14/min, and blood pressure is 165/105 mm Hg. He exhibits proximal muscle weakness. Laboratory findings include a fasting serum glucose level of 155 mg/dL, an 8:00 am serum cortisol level that is elevated at 54 μg/dL, and an elevated serum corticotropin level at 63 pg/mL. His serum cortisol level does not decrease with a low dose of dexamethasone, but decreases by 70% with a high dose. What test would be most helpful in establishing his diagnosis? □ (A) MRI of the brain □ (B) Abdominal CT scan of the adrenals □ (C) Serum assay for glycosylated hemoglobin □ (D) Biopsy of the gastrocnemius □ (E) Assay for urinary catecholamine metabolites

MRI of the brain (patient has Cushing disease; if the cortisol concentration and the corticotropin level increase, the pituitary is producing ACTH or there is an ectopic source such as a lung carcinoma; MRI to determine the size of the pituitary is the logical investigation in this case)

A 9-year-old girl has been bothered by right lower leg pain for the past 3 months. On physical examination, she is at the 150th percentile for height. She has a 4 × 5 cm, dark café-au-lait spot with a serpiginous border on her right hip, and a similar 3 × 6 cm spot on the posterior aspect of her right shoulder. She has significant breast and pubic hair development. A radiograph of the lower leg shows a 3-cm, circumscribed, lucent lesion involving the intramedullary right tibial diaphysis, with expansion of surrounding bone. A biopsy specimen of the lesion has the microscopic appearance shown in the figure. Abdominal CT scan shows bilateral adrenal enlargement. MRI of the brain shows a gadolinium-enhancing, 1.5-cm mass in the sella turcica. What is the most likely diagnosis? □ (A) Albers-Schönberg disease □ (B) Congenital adrenal hyperplasia □ (C) Maffucci syndrome □ (D) McCune-Albright syndrome □ (E) Ollier disease □ (F) Osteitis deformans

McCune-Albright syndrome (acquired somatic gene defect that occurs during embryogenesis and leads to excessive cyclic AMP production that fuels endocrine gland activity; growth hormone secreting pituitary adenomas, hyperthyroidism, adrenal hyperplasia, and sexual precocity can occur)

A 30-year-old man has had pain and burning on urination for the past week. On physical examination, he is febrile and has a pulse of 92/min, respirations of 18/min, and blood pressure of 80/45 mm Hg. Digital rectal examination indicates that he has an enlarged, tender prostate. There is costovertebral angle tenderness on the right. Scattered ecchymoses are present over the trunk and extremities. Laboratory studies show a blood culture positive for Klebsiella pneumoniae. The appearance of the RBCs in a peripheral blood smear is shown in the figure. These findings are most indicative of what condition? □ (A) Hereditary spherocytosis □ (B) Autoimmune hemolytic anemia □ (C) Microangiopathic hemolytic anemia □ (D) Iron deficiency anemia □ (E) Megaloblastic anemia

Microangiopathic hemolytic anemia (patient has Gram negative sepsis; widespread endothelial damage causes disseminated intravascular coagulation; fragmented RBCs and "helmet cells" are typical of conditions that can produce microangiopathic hemolytic anemia)

A 56-year-old man has noticed increasing abdominal girth and decreased libido for the past 7 months. Physical examination shows an enlarged abdomen with a fluid wave, but no tenderness or masses; the spleen tip is palpable. Bibasilar crackles are audible on auscultation of the chest. There is 1+ pitting edema to the knees. The testes are smaller than normal, but without masses. Laboratory findings show hemoglobin, 12.2 g/dL; hematocrit, 36.9%; MCV, 103 μm3; platelet count, 189,400/mm3; WBC count, 5762/mm3; Na+, 138 mmol/L; K+, 3.9 mmol/L; Cl−, 98 mmol/L; CO2, 24 mmol/L; creatinine, 1.1 mg/dL; glucose, 88 mg/dL; total protein, 5.7 g/dL; albumin, 2.7 g/dL; AST, 167 U/L; ALT, 69 U/L; alkaline phosphatase, 48 U/L; total bilirubin, 1.5 mg/dL; and prothrombin time, 23 seconds. What is the most likely diagnosis? □ (A) Adrenal atrophy □ (B) Aortic valvular stenosis □ (C) Autoimmune gastritis □ (D) Chronic glomerulonephritis □ (E) Hypertrophic cardiomyopathy □ (F) Micronodular cirrhosis

Micronodular cirrhosis (ascites, edema, and splenomegaly with lab evidence of hepatic dysfunction suggest hepatic disorder with portal hypertension; most common cause is hepatic cirrhosis from chronic alcohol abuse)

A 4-year-old girl has abrupt onset of vomiting, which remains protracted for 24 hours. On arrival at the emergency department, the child is lethargic and febrile to 37.7°C. The parents state that she had a mild upper respiratory tract illness 3 days ago, but was improving, and the only medication she received was acetylsalicylic acid (aspirin). On physical examination, there is poor skin turgor, the lungs are clear, the abdomen is nontender, and the heart rate is regular. Laboratory findings show Na+, 150 mmol/L; K+, 4.5 mmol/L; Cl−, 93 mmol/L; CO2, 30 mmol/L; glucose, 60 mg/dL; creatinine, 1.1 mg/dL; amylase, 25 U/L; AST, 386 U/L; ALT, 409 U/L; alkaline phosphatase, 120 U/L; total bilirubin, 1.1 mg/dL; ammonia, 80 μmol/L; and prothrombin time, 26 seconds. The child becomes comatose. What pathologic finding is most likely present in this patient? □ (A) Common bile duct atresia □ (B) Hepatic vein thrombosis □ (C) Hepatoblastoma □ (D) Intrahepatic duct lithiasis □ (E) Microvesicular steatosis □ (F) Multinucleated giant cells

Microvesicular steatosis (patient has microvesicular steatosis characteristic of Reye syndrome; caused by severe mitochondrial dysfunction in the brain and liver; biliary atresia with marked hyperbilirubinemia becomes apparent in the neonatal period)

A 38-year-old woman has had malaise and arthralgias for the past 14 months. Over the past month, she has developed areas of purpura on the distal extremities. On physical examination, her temperature is 37°C, pulse is 81/min, respirations are 14/min, and blood pressure is 140/90 mm Hg. She has scleral icterus and 1- to 3-cm areas of reddish purple discoloration on her skin. Several of these areas show focal ulceration. Laboratory findings show total protein, 7.1 g/dL; albumin, 3.3 g/dL; AST, 87 U/L; ALT, 95 U/L; alkaline phosphatase, 80 U/L; total bilirubin, 4 mg/dL; and direct bilirubin, 3.1 mg/dL. Serologic test results are positive for anti-HCV and negative for anti-HBs and IgM anti-HAV. Urinalysis shows 4+ proteinuria and 1+ hematuria. CT scan of the abdomen shows a small amount of ascites, mild hepatomegaly, and no splenomegaly or lymphadenopathy. A biopsy specimen of an ulcerated skin lesion shows leukocytoclastic vasculitis involving the upper dermis. What is the most likely diagnosis? □ (A) Autoimmune hemolytic anemia □ (B) Hepatocellular carcinoma □ (C) Hereditary hemochromatosis □ (D) Mixed cryoglobulinemia □ (E) Multiple myeloma

Mixed cryoglobulinemia (patients with HepC infection can develop chronic hepatitis with persistently elevated liver enzymes; some patients with HepC develop mixed cryoglobulinemia with a polyclonal increase in IgG; renal involvement is common; cryoglobulinemic vasculitis leads to skin hemorrhages and ulceration)

Over the past 2 years, a 44-year-old man has noticed a decline in dexterity of his right hand in his work as an auto mechanic. His right hand strength is weaker than the left. He experienced painful burning sensations in the left upper extremity 1 year ago. He has been bothered by decreased visual acuity in the left eye for the past month. He is insensitive to heat, and on taking a hot shower, his vision worsens. On physical examination, he is afebrile, and his blood pressure is normal. Motor strength in the right extremity is 4/5, but 5/5 elsewhere. Vision is 20/100 in the left eye and 20/40 in the right eye. One year later, he reports chronic constipation and urinary urgency, hesitancy, and incontinence. An MR image of the brain is shown in the figure. What is the most likely diagnosis? □ (A) Diabetes mellitus □ (B) Graves disease □ (C) HIV infection □ (D) Multiple sclerosis □ (E) Myasthenia gravis □ (F) Systemic lupus erythematosus

Multiple sclerosis (MR image shows white matter plaques of demyelination typical of MS; mean onset is 30 yrs; optic neuritis, weakness, and sensory changes are frequent manifestations)

A 44-year-old man has had worsening exercise tolerance and peripheral edema during the past 5 years. He has noted increasing central opacifications that interfere with vision. He has frontal baldness. During the past 2 years, he has had progressive memory loss with decreasing ability to perform activities of daily living. On physical examination, there is significant atrophy of masseter, temporalis, scalene, deltoid, trapezius, and sternocleidomastoid muscles. There is bilateral testicular atrophy. A 2-hour glucose tolerance test shows serum glucose of 156 mg/dL. Serum quantitative immunoglobulins show IgG of 450 mg/dL, IgA of 303 mg/dL, and IgM of 197 mg/dL. His condition worsens over the next 3 years, with increasing muscular weakness, and he dies of bronchopneumonia. An abnormality in which of the following gene products is most likely to be present? □ (A) α-1,4-Glucosidase □ (B) Dystrophin □ (C) Fibroblast growth factor receptor 3 □ (D) Mitochondrial oxidative phosphorylase □ (E) Myophosphorylase □ (F) Myotonin protein kinase □ (G) Neurofibromin

Myotonin protein kinase (patient has myotonic dystrophy; increased CTG repeat sequences in the gene; weakness in skeletal, cardiac, and smooth muscle develops; associated with cataracts, dementia, gonadal atrophy, and hypogammaglobulinemia)

A 76-year-old man has a history of difficulty voiding for the past 2 years, but now has had decreased urine output for the past 6 days. On examination, he is afebrile and normotensive. There is suprapubic tenderness on palpation. Laboratory studies show serum Na+, 150 mmol/L; K+, 4.5 mmol/L; Cl−, 111 mmol/L; CO2, 26 mmol/L; creatinine, 3 mg/dL; and glucose, 76 mg/dL. A urinary catheter is passed. After catheterization, diuresis ensues with polyuria. A urine sample of 1000 mL is collected. The urine sample contains 150 mEq of sodium and 60 mg of creatinine. Urinalysis shows pH 7; specific gravity 1.010; and no protein, glucose, nitrite, leukocyte esterase, or ketones. Which of the following conditions does he most likely have? □ (A) Acute cystitis □ (B) Autosomal dominant polycystic kidney disease □ (C) Chronic glomerulonephritis □ (D) Congestive heart failure □ (E) Nodular prostatic hyperplasia □ (F) Renal cell carcinoma

Nodular prostatic hyperplasia (findings are most characteristic for obstructive uropathy; function of both kidneys is affected, so the obstruction must be below the bladder neck; prostatic hyperplasia is common in older men)

A 49-year-old man has had increasing knee and hip pain for the past 10 years. The pain is worse at the end of the day. During the past year, he has become increasingly drowsy at work. His wife complains that he is a "world class" snorer. During the past month, he has experienced bouts of sharp, colicky, right upper abdominal pain. On physical examination, his temperature is 37°C, pulse is 82/min, respirations are 10/min, and blood pressure is 140/85 mm Hg. He is 175 cm (5 ft 8 in) tall and weighs 156 kg (body mass index 51). Laboratory findings show glucose of 139 mg/dL, total cholesterol of 229 mg/dL, and HDL cholesterol of 33 mg/dL. An arterial blood gas measurement shows pH of 7.3, Pco2 50 mm Hg, and Po2 70 mm Hg. What condition is most likely present? □ (A) Hashimoto thyroiditis □ (B) Hypertrophic cardiomyopathy □ (C) Laryngeal papillomatosis □ (D) Nonalcoholic steatohepatitis □ (E) Panlobular emphysema □ (F) Rheumatoid arthritis

Nonalcoholic steatohepatitis (patient is morbidly obese with complications including obesity hypoventilation syndrome, glucose intolerance, sleep apnea, cholelithiasis, and osteoarthritis)

A 41-year-old woman has had headaches with blurred vision for the past 3 days. Over the past day, she has developed increasing mental confusion. On admission to the hospital, her temperature is 37.9°C, pulse is 104/min, respirations are 25/min, and blood pressure is 70/40 mm Hg. On physical examination, she has petechial hemorrhages over her arms and trunk. A stool sample is positive for occult blood. Laboratory findings show hemoglobin of 9.1 g/dL, hematocrit of 27.2%, MCV of 92 μm3, RDW of 19%, platelet count of 8900/mm3, and WBC count of 8950/mm3. The peripheral blood smear shows schistocytes. A serum electrolyte panel shows Na+, 147 mmol/L; K+, 5 mmol/L; Cl−, 105 mmol/L; CO2, 26 mmol/L; creatinine, 3.3 mg/dL; urea nitrogen, 32 mg/dL; and glucose, 80 mg/dL. Ultra-large multimers of von Willebrand factor are present in plasma. What therapy should she receive emergently? □ (A) 2 U of packed RBCs □ (B) 6-pack of platelets □ (C) Dobutamine □ (D) Exploratory laparotomy □ (E) Plasmapheresis □ (F) Prednisone

Plasmapheresis (patient has thrombotic thrombocytopenic purpura with classic pentad of neurologic changes, fever, thrombocytopenia, microangiopathic hemolytic anemia, and decreased renal function; pathogenesis of TTP is related to release of von Willebrand factor)

A 7-year-old child has had worsening headaches and obtundation for the past 2 days. Physical examination shows temperature of 39.5°C, pulse of 103/min, respirations of 18/min, and blood pressure of 90/55 mm Hg. There is bilateral papilledema on funduscopic examination. No focal neurologic deficits are noted. Palpation of the abdomen reveals hepatosplenomegaly. Laboratory findings show hemoglobin, 9.5 g/dL; hematocrit, 28.8%; MCV, 101 μm3; platelet count, 145,000/mm3; WBC count, 6920/mm3; Na+, 146 mmol/L; K+, 5.5 mmol/L; Cl−, 106 mmol/L; CO2, 26 mmol/L; creatinine, 2.3 mg/dL; urea nitrogen, 22 mg/dL; LDH, 1095 U/L; and amylase, 45 U/L. The appearance of the peripheral blood smear is shown in the figure. What infectious agent is most likely to produce these findings? □ (A) Babesia microti □ (B) Borrelia burgdorferi □ (C) Leishmania donovani □ (D) Plasmodium falciparum □ (E) Trypanosoma gambiense □ (F) Wuchereria bancrofti

Plasmodium falciparum (child has cerebral malaria; smear shows numerous forms of the parasites in RBCs; falciparum malaria is the worst form; parasites released from cells at periodic intervals, leading to periodic fever and chills; parasites adhere to vascular endothelium and lead to ischemia in various organs including the brain; hemolytic anemia)

A 29-year-old woman has had fatigue with dizziness for the past 5 months. On physical examination, she has an erythematous malar rash. She has no lymphadenopathy, but there is a palpable spleen tip. She is afebrile. Laboratory studies show hemoglobin, 8.9 g/dL; hematocrit, 27.8%; MCV, 103 μm3; RBC distribution width index, 22; WBC count, 8650/mm3; platelet count, 222,000/mm3; and reticulocyte count, 3.3%. The peripheral blood smear shows polychromasia, but no schistocytes. Her serum total bilirubin is 3.2 mg/dL with direct bilirubin 0.8 mg/dL, and haptoglobin is 5 mg/dL. Antinuclear antibody and anti-double-stranded DNA tests are positive. What additional laboratory test finding is she most likely to have? □ (A) D-dimer 10 μg/mL □ (B) Increased RBC osmotic fragility □ (C) Positive Coombs test □ (D) Serum cobalamin (vitamin B12) 50 pg/mL □ (E) Serum ferritin 240 ng/mL

Positive Coombs test (patient has circulating antibody against her RBCs which leads to hemolytic anemia; Direct Coombs detects Ab bound to RBC; Indirect Coombs detects Ab in the plasma)

A case control study is performed involving persons diagnosed with "essential" hypertension. Genetic analysis reveals linkage disequilibrium. Haplotypes of affected persons differ from the controls in the chromosome containing the angiotensinogen gene. Which of the following types of genetic analysis is most likely to yield this information? □ (A) Fluorescence in situ hybridization □ (B) Giemsa banded karyotyping □ (C) RNA expression □ (D) Single nucleotide polymorphisms □ (E) Southern blotting

Single nucleotide polymorphisms (human genome is made of blocks of DNA with varying numbers of contiguous SNPs that form haplotypes and can cluster from linkage disequilibrium; similar haplotypes suggest shared inheritance)

A 51-year-old man has had increasing lethargy over the past year. On physical examination, his temperature is 37°C, pulse is 80/min, respirations are 15/min, and blood pressure is 145/90 mm Hg. He has deformity with increased size, and there is decreased range of motion of the first three metacarpophalangeal (MCP) joints on the right and the second two MCP joints on the left. There is a 2-cm, firm, painless nodule over the left olecranon bursa. A similar 1-cm nodule is palpated in the helix of the right ear, and another 1.5-cm nodule is palpable over the right Achilles tendon. Urinalysis shows specific gravity of 1.012, pH 5.5, 1+ hematuria, 1+ proteinuria, and no glucose. The serum urea nitrogen level is 31 mg/dL, and the creatinine is 3.2 mg/dL. The total serum cholesterol is 222 mg/dL. Aspiration of material from the nodule at the left elbow is performed. Which of the following types of crystals is most likely to be seen microscopically in this aspirate? □ (A) Calcium pyrophosphate dihydrate □ (B) Cholesterol □ (C) Cystine □ (D) Hydroxyapatite □ (E) Sodium urate

Sodium urate (findings are most typical of gouty arthritis; gout can lead to renal failure and to tophaceous deposits in soft tissues and joints; often accompanied by hyperlipidemia)

A 30-year-old man has fatigue from chronic hypochromic microcytic anemia. Hemoglobin electrophoresis reveals decreased Hgb A1 with increased Hgb A2 and F, due to imbalance of alpha and β globin chain synthesis. This results in RBC inclusions that cause membrane damage and accelerate apoptosis. Which of the following mutations is most likely to be present in the β globin gene of this man? □ (A) Trinucleotide repeat □ (B) New stop codon □ (C) Single base insertion, with frameshift □ (D) Splice site □ (E) Three base deletion

Splice site (this is one mechanism for β+ thalassemia; introns are usually involved, so flanking exons remain; some normal splicing can occur so some normal β globin chains are made)

A 29-year-old man has had a low-grade fever for the past 2 weeks. He has had increasing fatigue and a 2-kg weight loss during this time. On physical examination, his temperature is 37.5°C, pulse is 80/min, respirations are 17/min, and blood pressure is 150/70 mm Hg. His spleen tip is palpable, and there is left upper quadrant tenderness. There is bilateral costovertebral angle tenderness. A diastolic murmur is heard at the left sternal border. Subungual hemorrhages are noted on the digits of his hands. A needle track is present in the left antecubital fossa. Laboratory findings show hemoglobin, 13.6 g/dL; hematocrit, 41.8%; platelet count, 228,000/mm3; WBC count, 11,200/mm3; glucose, 66 mg/dL; AST, 101 U/L; ALT, 28 U/L; alkaline phosphatase, 89 U/L; amylase, 45 U/L; and total bilirubin, 0.9 mg/dL. Urinalysis shows 1+ hematuria, and WBCs and WBC casts. A chest radiograph shows a 3-cm nodule with an air-fluid level in the right upper lobe. What organism is most likely to be cultured from his blood? □ (A) Candida albicans □ (B) Cryptococcus neoformans □ (C) Escherichia coli □ (D) Listeria monocytogenes □ (E) Staphylococcus aureus □ (F) Streptococcus pyogenes □ (G) Yersinia enterocolitica

Staphylococcus aureus (findings are those of subacute bacterial endocarditis; needle track in left arm suggests IV drug use as the risk factor; individuals have right-sided and left-sided valvular lesions; vegetations are likely to embolize and the septic emboli can lead to infarction or infection)

A 70-year-old man has had increasing exercise intolerance and difficulty breathing for the past year. His family has noted memory loss and decreased ability to perform activities of daily living for the past 2 years. On physical examination, his temperature is 37.1°C, pulse is 70/min, respirations are 18/min, and blood pressure is 140/80 mm Hg. On auscultation of the chest, rales are audible in the lung bases, and there is a diastolic murmur. He has a marked decrease in sensation to light touch and pinprick over the lower extremities. His gait is ataxic, with the feet widely spaced. He cannot name any of three objects after 3 minutes. He thinks he is an astronaut returned from Mars. An echocardiogram shows aortic regurgitation with a widened aortic root and arch. MRI of the brain shows mild diffuse cortical atrophy and meningeal thickening. Infection with what organism would most likely produce these findings? □ (A) Borrelia burgdorferi □ (B) Coxsackievirus B □ (C) HIV □ (D) Mycobacterium leprae □ (E) Mycobacterium tuberculosis □ (F) Treponema pallidum □ (G) West Nile virus

Treponema pallidum (findings are consistent with tertiary syphilis; patient also has findings of syphilitic aortitis and neurosyphilis; positive VDRL rest result on CSF aids in diagnosis)

A 40-year-old man has noticed worsening myalgias and increasing difficulty swallowing over the past 2 years. When exposed to cold, the skin of his hands turns white. On physical examination, he has an erythematous rash extending across the bridge of his nose. There is swelling and warmth in the joints of his hands. Laboratory findings show hemoglobin, 12.2 g/dL; hematocrit, 36.5%; platelet count, 180,000/mm3; WBC count, 4510/mm3; serum glucose, 72 mg/dL; total bilirubin, 1 mg/dL; AST, 41 U/L; ALT, 19 U/L; alkaline phosphatase, 69 U/L; creatine kinase, 483 U/L; and creatinine, 1.3 mg/dL. The presence of antibodies to which of the following is most characteristic of his condition? □ (A) ANCA □ (B) Cyclic citrullinated peptide □ (C) Histone □ (D) Smith □ (E) Thyroid peroxidase □ (F) U1-RNP

U1-RNP (patient has mixed CT disease; has overlapping features of SLE, scleroderma, polymyositis, and rheumatoid arthritis)

A 5-year-old child who has received no medical care since birth has had gradual onset of markedly decreased vision bilaterally. The child also has a history of increased respiratory tract infections caused by Haemophilus influenzae, Streptococcus pneumoniae, Klebsiella pneumoniae, and rubeola. The figure shows the representative microscopic appearance of the bronchial mucosa. The child also has passed several urinary tract calculi. On physical examination, generalized papular dermatosis is noted. The child has xerophthalmia, and there is marked keratomalacia with corneal clouding. Bilateral crackles are audible in the lungs on auscultation. What disease process would most likely lead to these findings? □ (A) Cystic fibrosis □ (B) Congenital syphilis □ (C) HIV infection □ (D) Kartagener syndrome □ (E) Vitamin A deficiency

Vitamin A deficiency (leads to epithelial disorders affecting the cornea, skin, respiratory tract, and urinary tract)

A 56-year-old man has had increasing lower leg swelling during the past 6 months. Also, he has had so much difficulty breathing at night that he sleeps propped up on two pillows. On physical examination, his temperature is 37.1°C, pulse is 80/min, respirations are 17/min, and blood pressure is 110/70 mm Hg. On auscultation of the chest, bilateral crackles are audible at the lung bases. The liver span is increased. There is 2+ pitting edema to the thighs. Laboratory findings show hemoglobin, 13.4 g/dL; hematocrit, 40.2%; MCV, 88 μm3; platelet count, 229,300/mm3; and WBC count, 6715/mm3. He develops an acute psychosis 1 year later. He dies of aspiration pneumonia. At autopsy, there is anterior vermian atrophy and petechial hemorrhages with brown discoloration in the periaqueductal gray matter and shrunken mamillary bodies. A chronic deficiency of which of the following vitamins would most likely explain these findings? □ (A) Vitamin A (retinoic acid) □ (B) Vitamin B1 (thiamine) □ (C) Vitamin B2 (riboflavin) □ (D) Vitamin B3 (niacin) □ (E) Vitamin B12 (cobalamin) □ (F) Vitamin C (ascorbic acid) □ (G) Vitamin D (cholecalciferol) □ (H) Vitamin E (α-tocopherol)

Vitamin B1 (thiamine) (Patient has features of Wernicke disease and beriberi, but of which result from thiamine deficiency; can accompany chronic alcoholism)

A 14-year-old girl who has been in foster care for the past 11 years has not been with one caregiver for more than 1 year at a time. She is brought to the physician by the most recent caregiver, who obtained custody of the child 1 week ago. On physical examination, there are ecchymoses of the trunk, extremities, and gingiva. A hyperkeratotic, papular rash, with 0.4-cm lesions ringed by hemorrhage, is present in a similar distribution. The child has pain on movement of the arms and legs. There is abnormal depression of the sternum with prominence of the ribs and the costochondral junctions. Radiographs of the arms and legs show bowing of the long bones and widening of the metaphyses, with normal calcification. There is a right femoral subperiosteal hematoma. No fractures are noted. CBC shows hemoglobin of 10.8 g/dL, hematocrit of 32.4%, MCV of 77 μm3, platelet count of 201,300/mm3, and WBC count of 5730/mm3. What most likely explains these findings? □ (A) CFTR gene mutation □ (B) Inhibitor of procoagulant factor VIII □ (C) Collagen gene mutation □ (D) Vitamin C deficiency □ (E) Multiple blunt trauma

Vitamin C deficiency (Vitamin C deficiency is manifested by a decrease in synthesis of collagen peptides from inadequate hydroxylation of procollagen; affects bone matrix formation, vascular integrity, and epithelial function; bleeding into joints and soft tissues with minimal trauma)

An ultrasound examination of a 29-year-old primigravida performed at 18 weeks' gestation shows a male fetus that is mildly growth retarded. Multiple congenital anomalies are present, including ventricular and atrial septal defects, horseshoe kidney, and omphalocele. Amniocentesis is performed, and the fetal cells obtained are examined using FISH analysis. Based on the findings shown in the figure, which of the following karyotypic abnormalities is most likely to be present in this fetus? □ (A) 46,XY,der(14;21)(q10.0),+21 □ (B) 47,XY,+18 □ (C) 47,XXY □ (D) 46,XY,del(22q11) □ (E) 45,X/46,XX

47,XY,+18 (Infant has trisomy 18 findings)

A boy born to a 40-year-old woman, who had an uncomplicated pregnancy, is noted at birth to be at the 70th percentile for height and weight. On physical examination, the infant has bilateral palmar transverse creases and absent distal flexion creases on the fifth digits. The palpebral fissures are oblique. He has brachycephaly. On auscultation of the chest, a holosystolic murmur is audible. During childhood, mental retardation is exhibited, but the child is able to perform activities of daily living. At age 17, the boy has a series of severe upper respiratory tract infections. CBC shows hemoglobin, 10.2 g/dL; hematocrit, 30.5%; MCV, 89 μm3; platelet count, 103,000/mm3; and WBC count, 19,200/mm3 with 14% segmented neutrophils, 6% bands, 22% lymphocytes, 13% monocytes, and 45% blasts. What karyotype is most likely to be present in this boy? □ (A) 45,X □ (B) 46,XY □ (C) 47,XY,+13 □ (D) 47,XY,+18 □ (E) 47,XY,+21 □ (F) 47,XXY □ (G) 69,XYY

47,XY,+21 (child has Down syndrome; has developed leukemia as a complication)

A 39-year-old man goes to the physician because he has experienced diminished libido for the past 4 months. Review of systems indicates that he has had frequent headaches over the past 2 months. On physical examination, he has gynecomastia bilaterally, normal-sized testes in the scrotum, and difficulty with peripheral vision. His visual acuity is 20/20 bilaterally. Laboratory findings show Na+, 141 mmol/L; K+, 4.1 mmol/L; Cl−, 102 mmol/L; CO2, 25 mmol/L; glucose, 75 mg/dL; and creatinine, 1.2 mg/dL. Which of the following neoplasms is most likely to be diagnosed? □ (A) Adenohypophyseal adenoma □ (B) Carcinoid tumor □ (C) Medullary carcinoma □ (D) Pheochromocytoma □ (E) Renal cell carcinoma □ (F) Small-cell anaplastic carcinoma

Adenohypophyseal adenoma (patient has hyperprolactinemia caused by a prolactinoma; macroadenomas produce homonymous hemianopsia and can secrete prolactin to cause gynecoastia)

As a student in the health sciences, you have just finished this review book, learning to apply your knowledge base in pathology to clinical and experimental scenarios of human disease states. Which of the following represents your best application of this knowledge? □ (A) Collegial consultation □ (B) Compassionate care □ (C) Differential diagnosis □ (D) Lifelong learning □ (E) Patient education □ (F) Research projects

All are correct! You win

A 50-year-old woman has a 20-year history of Crohn disease. Over the past 14 months, she has had increasing fatigue with worsening peripheral edema. On physical examination, she has pitting edema to her knees. Laboratory studies show hemoglobin, 13.2 g/dL; hematocrit, 40.1%; platelet count, 203,000/mm3; and WBC count, 7470/mm3. Serum urea nitrogen is 35 mg/dL, and creatinine is 3.8 mg/dL. Urinalysis shows proteinuria without hematuria, glucosuria, or ketonuria. A renal biopsy specimen shows deposits of amorphous pink material in glomeruli, arterioles, and peritubular interstitium. By electron microscopy, these pink deposits are composed of non-branching 7.5- to 10-nm fibrils. Which of the following proteins is most likely to form these fibrils? □ (A) Amyloid-associated □ (B) Amyloid precursor protein □ (C) Atrial natriuretic peptide □ (D) β2-microglobulin □ (E) Calcitonin □ (F) Lambda light chain □ (G) Transthyretin

Amyloid-associated (patient has reactive systemic amyloidosis from her underlying granulomatous ileitis and serum amyloid associated protein is generated by chronic inflammation)

A 35-year-old man presents with increasing breathlessness on exertion. On physical examination his height is 208 cm (82 in) with long arms and tapering fingers. His joints are exceptionally flexible. He also has bilateral subluxation of crystalline lenses. Echocardiography reveals mitral valve prolapse and an aneurysm in his ascending aorta. Which of the following experimental therapies is likely to be beneficial in treating this man? □ (A) Anti-TGF-β antibody □ (B) Bone marrow transplantation □ (C) Low-calorie, low-cholesterol diet □ (D) Lysosomal stabilizing agents □ (E) Transduction of fibrillin-1 gene into T cells

Anti-TGF-β antibody (patient has features of Marfan syndrome; primary defect is mutation in fibrillin-1 gene, which causes poor formation of microfibrils; secondary effect on transforming growth factor-beta bioavailability, which is normally sequestered by microfibrils; in marfans there isn't enough microfibrils to sequester the growth factor so it causes cardiac symptoms)

A 41-year-old woman has had increasing lethargy and weakness over the past 3 years. She complains of being cold most of the time and wears a sweater in the summer. One year ago, she had menorrhagia, but now she has oligomenorrhea. She has difficulty concentrating, and her memory is poor. She has chronic constipation. On physical examination, her temperature is 35.5°C, pulse is 54/min, respirations are 13/min, and blood pressure is 110/70 mm Hg. She has alopecia, and her skin appears coarse and dry. Her face, hands, and feet appear puffy, with a doughlike consistency to the skin. Laboratory findings show hemoglobin, 13.8 g/dL; hematocrit, 41.5%; AST, 26 U/L; ALT, 21 U/L; total bilirubin, 1 mg/dL; Na+, 140 mmol/L; K+, 4.1 mmol/L; Cl−, 99 mmol/L; CO2, 25 mmol/L; glucose, 73 mg/dL; and creatinine, 1.1 mg/dL. Which of the following serologic test findings is most likely to be positive? □ (A) Anticentromere antibody □ (B) Anti-DNA topoisomerase antibody □ (C) Antimitochondrial antibody □ (D) Antinuclear antibody □ (E) Antiribonucleoprotein antibody □ (F) Anti-thyroid peroxidase antibody

Anti-thyroid peroxidase antibody (patient has hypothyroidism; one of the most common causes is Hashimoto thyroiditis, which eventually leads to thyroid atrophy; antimicrosomal and antithyroglobulin Ab can be seen in Hashimoto thyroiditis)

A 51-year-old man living on the island of St. Helena has had a downturn in his political fortunes. Over the past 3 years, and particularly over the past year, he has had increasing bouts of abdominal pain, anorexia, nausea, vomiting, dysuria, lethargy, spiking fevers, diarrhea, constipation, excessive weakness, heavy perspiration, and weight loss. He is given a large dose of calomel (a mercury-containing compound) a few days before his death May 5, 1821, a treatment that has since vanished for good reason. An autopsy shows hepatomegaly (with steatosis?) and ulceration with thickening of the stomach. The autopsy report does not record skin and nail changes, such as hyperkeratosis and hyperpigmentation. If those changes had been present, and squamous cell carcinoma of the skin, the findings would have been most suggestive of chronic poisoning with which of the following metals? □ (A) Arsenic □ (B) Beryllium □ (C) Chromium □ (D) Cobalt □ (E) Lead □ (F) Nickel

Arsenic (one hypothesis regarding the death of Napoleon Bonaparte is that he was the victim of arsenic poisoning perpetrated by former enemies; Chronic arsenic exposure is associated with elevated risk of skin cancer and cancers of the lung, liver, bladder, kidney, and colon)

A 52-year-old woman has noticed increasing thirst and urine output for the past 6 months. She has become more lethargic and has had decreased mental agility during that time. She has had flank pain on the right during the past month. On physical examination, her temperature is 37°C, pulse is 77/min, respirations are 14/min, and blood pressure is 150/95 mm Hg. There are bilateral palpable masses in the abdomen. The figure shows an abdominal CT scan. Urinalysis shows specific gravity of 1.010, pH 6.5, 2+ proteinuria, 2+ hematuria, and no glucose or ketones. Laboratory findings show hemoglobin of 10.4 g/dL, hematocrit of 31.3%, glucose of 102 mg/dL, AST of 30 U/L, ALT of 21 U/L, creatinine of 5.5 mg/dL, and urea nitrogen of 53 mg/dL. One year later, she develops a sudden, severe headache. CT scan of the head shows a subarachnoid hemorrhage at the base of the brain. What is the most likely diagnosis? □ (A) Adult-onset medullary cystic disease □ (B) Cystinosis □ (C) Diabetes mellitus type 2 □ (D) Autosoma-dominant polycystic kidney disease □ (E) Polyarteritis nodosa □ (F) Wegener granulomatosis □ (G) Wilson disease

Autosoma-dominant polycystic kidney disease (10% of individuals with ADPKD have a berry aneurysm of the circle of Willis; may be complicated by rupture and hemorrhage into the subarachnoid space)

A 60-year-old man has pain in his hands and feet on arising in the morning. The pain persists for almost 2 hours and is aggravated by movement. He would rather stay in bed. He has weakness, easy fatigability, and anorexia, and reports a 6-kg weight loss over the past 2 years. On physical examination, his temperature is 37.4°C, pulse is 74/min, respirations are 20/min, and blood pressure is 110/70 mm Hg. There is warmth, tenderness, and limitation of motion of the joints of his hands. He develops joint deformities over the next 5 years. A chest radiograph shows extensive interstitial lung disease and a prominent right-sided heart border. Spirometry reveals decreased FEV1 and FVC. Additional findings include 1-cm, firm, subcutaneous nodules over the olecranon bursae and Achilles tendons. What is the most likely diagnosis? □ (A) Caplan syndrome □ (B) Carney syndrome □ (C) Churg-Strauss syndrome □ (D) Felty syndrome □ (E) Hamman-Rich syndrome □ (F) Kartagener syndrome □ (G) Trousseau syndrome

Caplan syndrome (includes pneumoconiosis coupled with rheumatoid arthritis)

A 68-year-old man has had worsening dyspnea and orthopnea for the past 3 years with increased production of frothy sputum. On examination, crackles are auscultated at lung bases. A chest radiograph shows bilateral interstitial infiltrates, distinct Kerley B lines, and a prominent left heart border. Laboratory studies show Na+, 135 mmol/L; K+, 3.8 mmol/L; Cl−, 99 mmol/L; CO2, 25 mmol/L; glucose, 76 mg/dL; creatinine, 1.5 mg/dL; and urea nitrogen, 30 mg/dL. Fractional excretion of sodium is less than 1%. Plasma renin, aldosterone, and antidiuretic hormone levels all are increased. Brain-type natriuretic peptide (BNP) is 200 pg/mL (normal <100 pg/mL). What is the most likely diagnosis? □ (A) Addison disease □ (B) Chronic glomerulonephritis □ (C) Congestive heart failure □ (D) Conn syndrome □ (E) Pulmonary fibrosis □ (F) SIADH

Congestive heart failure (patient has left-sided heart failure with pulmonary edema and congestion; other options don't explain pulmonary edema)

A 29-year-old man has been bothered by diarrhea for the past month. He has about 5 to 10 stools per day that are watery and nonbloody. Physical examination shows he is afebrile. Bowel sounds are present, and there is mild diffuse abdominal pain on palpation in the region of the umbilicus. His stool is negative for occult blood, ova, and parasites. His hemoglobin is 11.1 g/dL, hematocrit is 34.5%, MCV is 120 μm3, platelet count is 286,300/mm3, and WBC count is 7670/mm3. His serum cobalamin is 105 pg/mL, and folate is 28 ng/mL. d-Xylose absorption is normal. Quantitative stool fat is normal. His serum total bile acids are decreased. Serologic testing reveals antibodies to Saccharomyces cerevisiae, but no antineutrophil cytoplasmic autoantibodies or antiendomysial antibodies. A hydrogen breath test is normal. Which of the following conditions does he most likely have? □ (A) Atrophic gastritis □ (B) Celiac disease □ (C) Crohn disease □ (D) Diverticulosis □ (E) Lactase deficiency □ (F) Pancreatitis □ (G) Shigellosis □ (H) Ulcerative colitis

Crohn disease (malabsorption is primarily in the terminal ileum, based on the decreased cobalamin, which leads to pernicious anemia and decreased enterohepatic circulation of bile acids; Crohn is favored over UC from the serologic profile)

A 31-year-old woman has experienced menorrhagia for the past year. She has developed easy bruisability to her skin, even with minor trauma, over the past month. She has multiple contusions of varying ages seen over her torso and extremities on physical examination. She has no difficulty with movement. No neurologic deficits are noted. Prothrombin time is 12.5 seconds, and partial thromboplastin time is 27 seconds. Hemoglobin is 13.3 g/dL, hematocrit is 42.1%, MCV is 95 μm3, platelet count is 25,000/mm3, and WBC count is 7500/mm3. A bone marrow biopsy specimen shows increased numbers of megakaryocytes. What is the most likely diagnosis? □ (A) Acute promyelocytic leukemia □ (B) Antiphospholipid syndrome □ (C) Hemophilia A □ (D) Immune thrombocytopenic purpura □ (E) Von Willebrand disease

Immune thrombocytopenic purpura (likely diagnosis given the low platelet count and on other cytopenias or coagulation disorders on the list)

The prenatal course of a 25-year-old primigravida is uncomplicated. She gives birth to a 4500-g boy whose Apgar scores are 8 and 10 at 1 minute and 5 minutes. Shortly after birth, he develops irritability with seizure activity. On examination, the infant is normally developed with no anomalies. The lungs are clear to auscultation. Laboratory studies show serum Na+, 145 mmol/L; K+, 4.2 mmol/L; Cl−, 99 mmol/L; CO2, 25 mmol/L; urea nitrogen, 0.4 mg/dL; and glucose, 18 mg/dL. Which of the following pathologic findings is most likely to be present in the pancreas of this infant? □ (A) Acute pancreatitis □ (B) Amyloid deposition □ (C) Adenocarcinoma □ (D) Chronic pancreatitis □ (E) Fatty replacement □ (F) Insulitis □ (G) Islet hyperplasia □ (H) Pseudocyst

Insulitis (maternal diabetes can result in hyperplasia of the fetal islets because of the maternal hyperglycemic environment; after birth the hyperplastic islets continue to overfunction, which results in neonatal hypoglycemia; infants of diabetic mothers also tend to exhibit macrosomia because of the growth-promoting effectsof increased insulin levels)

A 21-month-old child has had recurrent otitis media complicated by mastoiditis for the past 3 months. On physical examination, there is a seborrheic eruption on the skin of the trunk and scalp. Hepatosplenomegaly and generalized non-tender lymphadenopathy are present. A chest radiograph shows bilateral 0.5- to 2-cm pulmonary nodules, and there is a 1-cm lesion on the right clavicle and a 1.5-cm lesion on the left seventh rib, both osteolytic. Laboratory findings show hemoglobin of 10.4 g/dL, hematocrit of 31.2%, platelet count of 93,400/mm3, WBC count of 5780/mm3, glucose of 72 mg/dL, and creatinine of 0.5 mg/dL. A bone marrow biopsy specimen shows reduced hematopoiesis with increased numbers of large cells having oval vesicular nuclei and vacuolated cytoplasm that mark immunocytologically for CD1a. What is the most likely diagnosis? □ (A) Acute lymphoblastic leukemia □ (B) Gaucher disease □ (C) Langerhans cell histiocytosis □ (D) Leishmaniasis □ (E) Multiple myeloma □ (F) Myelodysplastic syndrome

Langerhans cell histiocytosis (child has Letterer-Siwe disease; acute disseminated form of Langerhans cell histiocytosis)

A 54-year-old man has had nausea for the past 6 months, but he does not report hematemesis. He has increasing malaise. On physical examination, he has decreased sensation to pinprick and light touch over the lower extremities bilaterally. He exhibits mild ataxia when walking. An upper gastrointestinal endoscopy study shows the absence of gastric rugal folds, but no ulceration or mass. Which of the following laboratory findings is most likely to be reported? □ (A) Positive anti-Smith antibody □ (B) Deficiency of factor V □ (C) Positive Helicobacter pylori antibody □ (D) MCV 125 μm3 □ (E) Urine glucose 4+

MCV 125 μm3 (patient has pernicious anemia owing to atrophic gastritis with lack of intrinsic factor to bind dietary vitamin B12 for absorption; condition leads to megaloblastic anemia and subacute combined spinal cord degeneration of dorsal and lateral tracts)

A 24-year-old woman has developed right-sided facial pain over the past 24 hours. During that time, she has become lethargic and obtunded. Her medical history shows a 5-kg weight loss over the past 6 months, despite increasing caloric intake. On physical examination, there is swelling with marked tenderness over the right maxilla, exophthalmos on the right side, diffuse abdominal pain, poor skin turgor, and dry mucous membranes. Her temperature is 37.7°C. She has tachycardia, but no murmurs, and tachypnea; the lung fields are clear. Laboratory findings show hemoglobin, 13.1 g/dL; hematocrit, 39.4%; Na+, 131 mmol/L; K+, 4.6 mmol/L; Cl−, 92 mmol/L; CO2, 9 mmol/L; glucose, 481 mg/dL; and creatinine, 1 mg/dL. An arterial blood gas measurement shows pH 7.2, Po2 98 mm Hg, Pco2 28 mm Hg, and HCO3− 10 mmol/L. Fine needle aspiration of the right maxillary region is performed. What organism is most likely to be present in this aspirate? □ (A) Actinomyces israelii □ (B) Bacillus anthracis □ (C) Cytomegalovirus □ (D) Clostridium perfringens □ (E) Cryptococcus neoformans □ (F) Mucor circinelloides □ (G) Pseudomonas aeruginosa

Mucor circinelloides (patient has type 1 DM with ketoacidosis, which is the setting for infection by Mucor in the paranasal sinuses)

A 12-year-old boy develops fever, accompanied by occasional headaches, malaise, fatigue, and nausea after being bitten by a dog. One day later, he experiences episodes of rigidity, hallucinations, breath holding, and difficulty swallowing because of uncontrollable oral secretions. Dr. Louis Pasteur is called upon. "The death of this child appearing to be inevitable, I decided, not without lively and sore anxiety, as may well be believed, to try ... the method which I had found constantly successful with dogs. Consequently, 60 hours after the bites [the child] was inoculated under a fold of skin with half a syringeful of the spinal cord of a rabbit. In the following days, fresh inoculations were made. I thus made 13 inoculations." The boy survived. Which of the following pathologic findings is most characteristic of the boy's disease? □ (A) Anterior horn cell loss □ (B) Gummatous necrosis □ (C) Multinucleated giant cells □ (D) Necrotizing vasculitis □ (E) Negri bodies □ (F) Pseudocysts with bradyzoites □ (G) Spongiform encephalopathy

Negri bodies (patient has infectious disease in which the virus travels from the site of an infected animal bite up the nerves to the CNS; pathognomonic Negri bodies are found in the hippocampal pyramidal cells and cerebellar Purkinje cells)

A 64-year-old man develops a low-volume mucoid diarrhea. He has had about five bowel movements per day, accompanied by cramping abdominal pain, for the past 2 months. The stool is occasionally blood-streaked. On physical examination, he appears pale. Vital signs include temperature of 37.2°C, pulse of 84/min, respirations of 15/min, and blood pressure of 115/75 mm Hg. A colonoscopy is performed; the figure shows a representative image of the mucosa from the rectum to the lower portion of the sigmoid. The remaining colonic mucosa appears normal. Biopsy specimens of the affected colon show mucosal crypt distortion, focal crypt abscesses, and mixed inflammatory infiltrates extending to the lamina propria. Over the next 2 years, the patient develops polyarthritis with no joint deformity, and uveitis and chronic active hepatitis. Which of the following inflammatory diseases is he most likely to develop? □ (A) Dermatitis herpetiformis □ (B) Rheumatoid arthritis □ (C) Orchitis □ (D) Thyroiditis □ (E) Primary sclerosing cholangitis □ (F) Atrophic gastritis

Primary sclerosing cholangitis (UC has a peak age of onset between 15 and 30 yrs with second peak between 60 and 80 yrs; colonoscopic appearance is of mucosal erythematous and finely granular surface that looks like sandpaper; primary sclerosing cholangitis, uveitis, migratory polyarthritis, and erythema nodosum are common to UC)

An 18-year-old woman has had recurrent acute attacks of dyspnea for the past 10 years. Between these attacks, she has no medical problems. She is brought to the emergency department within an hour of onset of the latest episode. On physical examination, her temperature is 37.4°C, pulse is 110/min, respiratory rate is 18/min, and blood pressure is 110/70 mm Hg. Expiratory wheezes are auscultated over the chest bilaterally. A chest x-ray shows bilateral radiolucency with no masses or infiltrates. Pulmonary function studies show severe limitation of airflow, which is relieved on injection of epinephrine. Sputum cytologic examination shows abundant mucus with an inflammatory infiltrate dominated by eosinophils, but mixed with neutrophils and macrophages. Which of the following immunological mechanisms is of primary importance in the pathogenesis of her disease? □ (A) Activation of neutrophils and macrophages by IL-8 □ (B) Chemoattraction of eosinophils by exotoxin □ (C) Proliferation of the TH2 subset of CD4+ T cells □ (D) Recruitment of monocytes by interferon-γ □ (E) Stimulation of bronchial smooth muscle cells by ADAM-33

Proliferation of the TH2 subset of CD4+ T cells (patient has classic history for atopic bronchial asthma that usually begins in childhood; disease symptoms are related to type 1 hypersensitivity but it is fundamentally T cell mediated chronic inflammation of the bronchial wall; TH2 CD4+ T cells drive type 1 hypersensitivity by favoring IgE production and eosinophil recruitment)

A 35-year-old woman has had a low-grade fever and worsening nonproductive cough and dyspnea for the past 2 years. On examination, she has breath sounds in all lung fields. A chest radiograph shows prominent hilar lymphadenopathy and increased interstitial lung markings. An arterial blood gas shows pH 7.45, Po2 83 mm Hg, Pco2 30 mm Hg, and HCO3 − 20 mEq/L. Pulmonary function tests show total lung capacity 3 L (60% of predicted), FEV1 2.5 (66% of predicted), and DLCO 10 mL/min/mm Hg (50% of predicted). Her pulmonary compliance is reduced. What is the most likely diagnosis? □ (A) α1-antitrypsin deficiency □ (B) Adult respiratory distress syndrome □ (C) Chronic bronchitis □ (D) Goodpasture syndrome □ (E) Nonatopic asthma □ (F) Sarcoidosis

Sarcoidosis (patient has restrictive lung disease; adenopathy is characteristic for sarcoid)

A 47-year-old woman has had increasing abdominal enlargement, with no significant pain, and diarrhea for the past 3 months. She goes to the physician, who performs a cursory physical examination and obtains a stool culture that is negative. The diagnosis is irritable bowel syndrome. She continues to have increasing abdominal enlargement over the next month, reaching the size of a 5-month pregnancy. She sees another physician, who finds a fluid wave on examination of the abdomen. Abdominal CT scan shows massive ascites and scattered 0.5- to 1.5-cm cystic to solid nodules on the surfaces of the bowel and abdominal wall. Paracentesis yields a yellow, slightly cloudy fluid with a high protein content. Cytologic examination of the fluid shows clusters of malignant cells. Laboratory studies show a positive CA 125 and a negative carcinoembryonic antigen test result. What is the most likely neoplasm? □ (A) Adenocarcinoma of the ileum □ (B) Carcinoid tumor □ (C) Endometrioid carcinoma □ (D) Malignant mesothelioma □ (E) Serous cystadenocarcinoma □ (F) Mucinous cystadenoma

Serous cystadenocarcinoma (pattern of metastases with seeding of the peritoneal cavity along with the microscopic and serum tumor markers are most characteristic for an ovarian serous cystadenocarcinoma; most common malignant neoplasm arising in the ovary)

A 4-month-old boy was born at term to an 18-year-old woman, G1, P0, after a normal pregnancy. The woman returned home from work one evening and was told by her boyfriend, who is staying at her home, that the infant died suddenly. An autopsy shows no external anomalies. The infant's height and weight are at the 40th percentile. Internal examination reveals subarachnoid hemorrhage at the vertex and subdural hemorrhage over the right parietal lobe. The right eye shows petechial hemorrhages at the ora serrata. There is a soft-tissue hemorrhage in the right upper arm. There is a recent fracture of the occipital bone. A section of the frontal cortex shows axonal retraction balls in white matter tracts. What is the most likely diagnosis? □ (A) α-Thalassemia □ (B) Congenital syphilis □ (C) Edwards syndrome □ (D) Hemophilia A □ (E) Osteogenesis imperfecta □ (F) Shaken baby syndrome □ (G) Sudden infant death syndrome □ (H) Thanatophoric dysplasia

Shaken baby syndrome (sudden nature of death requires investigation by medical examiner; pattern of injuries is consistent with vigorous shaking of the infant)

Within the past 24 hours, a 26-year-old, previously healthy woman has developed a high fever and generalized diffuse erythematous macular rash resembling a sunburn. She is menstruating, and her menstrual cycles are regular. She has nausea, vomiting, abdominal pain, diarrhea, myalgias, sore throat, headache, and dizziness. On physical examination, her temperature is 39.4°C, pulse is 101/min, respirations are 18/min, and blood pressure is 90/40 mm Hg. She has oropharyngeal and conjunctival hyperemia. The vaginal mucosa is erythematous. A tampon is present in the vaginal vault. She is disoriented, but there are no neurologic deficits. Laboratory findings show hemoglobin, 13.5 g/dL; hematocrit, 41.4%; platelet count, 100,000/mm3; WBC count, 11,200/mm3; glucose, 70 mg/dL; creatinine, 2.5 mg/dL; total bilirubin, 2.4 mg/dL; AST, 82 U/L; and ALT, 29 U/L. A chest radiograph shows no abnormal findings. She receives supportive therapy of nafcillin with clindamycin and improves, but skin and mucous membrane desquamation is noted 10 days later. These findings are most likely produced by an exotoxin elaborated by which of the following organisms? □ (A) Bacillus anthracis □ (B) Clostridium perfringens □ (C) Enterococcus □ (D) Listeria monocytogenes □ (E) Staphylococcus aureus □ (F) Vibrio cholerae

Staphylococcus aureus (patient has toxic shock syndrome; some strains of staph aureus produce exotoxins which act as superantigens and provoke an exuberant and dysregulated immune response characterized by release of cytokines that mediate cell injury)

A family pedigree reveals first and second-generation female relatives with premature menopause and male relatives with a progressive neurodegenerative disorder starting by their sixth decade. There are more males than females exhibiting mental retardation from childhood by the fourth generation. Genetic analysis of affected persons reveals CGG repeat expansions in a gene encoding for a protein that binds mRNA transcripts in neurons and shuttles them to the synapses. An abnormality involving which of the following organs is most likely to be present in affected males? □ (A) Adrenal □ (B) Pancreas □ (C) Pituitary □ (D) Testis □ (E) Thyroid

Testis (family has fragile-X syndrome; anticipation occurred with premutations of limited triple repeat expansions present in the first two generations, while later generations had full mutations with larger expansions)

Genetic testing of a family is performed using Southern blot with a DNA probe spanning the region of a suspected mutation. The results are shown in the figure. Based on these findings, which of the following types of gene mutations is most likely to be present in this family? □ (A) Frameshift □ (B) Nonsense (stop codon) □ (C) Point □ (D) Three-base pair deletion □ (E) Trinucleotide repeat

Trinucleotide repeat (this is the inheritance pattern for fragile X syndrome which is caused by triple repeat expansions in the FMR1 gene; repeats accumulate until symptomatic disease presents)

A 19-year-old man has been having headaches for the past month. On physical examination, his temperature is 37°C, pulse is 77/min, respirations are 14/min, and blood pressure is 160/95 mm Hg. On funduscopic examination, there are bilateral retinal angiomas. Abdominal CT scan shows a 3-cm mass involving the right adrenal gland. Laboratory testing shows increased urinary catecholamines. The mass is removed surgically. He develops a movement disorder 5 years later with incoordination and ataxia. MRI of the brain shows a 2-cm mass in the left cerebellar hemisphere and a 1-cm mass in the vermis. These are removed surgically. Six years later, he has right flank pain with hematuria; his abdominal CT scan is shown in the figure. His hemoglobin concentration is 20.3 g/dL, and hematocrit is 60.9%. Which of the following gene mutations and associated neoplasms does he most likely have? □ (A) WT-1—Beckwith-Wiedemann syndrome □ (B) MET—Denys-Drash syndrome □ (C) APC—Gardner syndrome □ (D) NF2—Neurofibromatosis type 2 □ (E) TSC-1—Tuberous sclerosis □ (F) VHL—von Hippel-Lindau disease

VHL—von Hippel-Lindau disease (rare; results from acquired or inherited mutation in a tumor suppressor gene)

A 12-year-old boy has had worsening problems with joint mobility involving his arms and legs, particularly his knees and ankles, for the past 6 years. He has been receiving therapy for this condition. His grandfather had a similar condition and died at age 25. On physical examination, he has no visible petechiae or areas of purpura. Laboratory studies show that prothrombin time is 12 seconds, and partial thromboplastin time is 52 seconds. After addition of an equivalent aliquot of normal plasma, the partial thromboplastin time is 30 seconds. Hemoglobin is 12.9 g/dL, platelet count is 238,500/mm3, and WBC count is 6620/mm3. His platelet function studies are normal. What is the most likely inheritance pattern for his condition? □ (A) Confined placental mosaicism □ (B) Autosomal recessive mutation □ (C) Autosomal dominant mutation □ (D) X-linked mutation □ (E) Germline mutation

X-linked mutation (patient most likely has hemophilia A, which is monitored with factor VII levels)

A 39-year-old man experiences sudden onset of a severe headache. Physical examination shows no localizing neurologic signs and no organomegaly. A stool sample is positive for occult blood. Areas of purpura appear on the skin of his extremities. Laboratory studies show hemoglobin of 9.6 g/dL, hematocrit of 28.9%, platelet count of 26,400/mm3, and WBC count of 75,000/mm3. The peripheral blood smear has the appearance shown in the figure; schistocytes also are seen. The plasma D-dimer level (fibrin degradation products), prothrombin time, and partial thromboplastin time all are elevated. Cytogenetic analysis of cells from a bone marrow biopsy specimen is most likely to yield what karyotypic abnormality? □ (A) t(8;21) □ (B) t(9;22) □ (C) t(14;18) □ (D) t(15;17) □ (E) t(8;14)

t(15;17) (peripheral blood smear is characteristic of acute promyelocytic leukemia/AML M3; many promyelocytes that contain prominent azurophilic granules and short, red, cytoplasmic, rodlike inclusions called Auer rods; release of granules can trigger coagulation cascade and cause DIC)

A 16-year-old girl has had a history of fatigue and weakness for her entire life. She has not undergone puberty. On physical examination, secondary sex characteristics are not well developed. She has hepatosplenomegaly. CBC shows hemoglobin of 9.1 g/dL, hematocrit of 26.7%, MCV of 66 μm3, platelet count of 89,000/mm3, and WBC count of 3670/mm3. The appearance of the peripheral blood smear is shown in the figure. Additional laboratory findings include serum glucose of 144 mg/dL, TSH of 6.2 mU/mL, and ferritin of 679 ng/mL. A mutation involving which of the following genes is most likely to be present in this girl? □ (A) AAT □ (B) β-Globin □ (C) CFTR □ (D) G6PD □ (E) HFE □ (F) NADPH oxidase □ (G) Ankyrin

β-Globin (patient has β-thalassemia; decreased β-Globin chain formation with increased Hb A2 and F to compensate; ineffective erythropoiesis and increased erythropoietin to drive increased iron absorption, leading to iron overload)


संबंधित स्टडी सेट्स

Combo with "Chapter 18" and 1 other

View Set

PSYC 2600: Chapter 13: Prejudice

View Set